Deck 2: Behavioral science

Full screen (f)
exit full mode
Question
A 46-year-old man is admitted to the hospital for atypical chest pain.  His medical history is significant for hypertension controlled with amlodipine and hypercholesterolemia treated with atorvastatin; he has a family history of depression and heart disease.  During review of his substance use history, the patient says that he has been a "regular drinker" for the last 5 years.  He drinks a 6-pack of beer every night and has 2 more beers in the morning to help him "get through the day."  He also admits to smoking marijuana occasionally but does not use tobacco or other illicit drugs.  Which of the following symptoms or signs is most likely to appear earliest during this patient's hospitalization?

A)Fluctuating arousal level
B)Hypersomnolence
C)Nystagmus
D)Tonic-clonic seizures
E)Tremulousness
F)Visual hallucinations
Use Space or
up arrow
down arrow
to flip the card.
Question
A 32-year-old man is diagnosed with major depressive disorder and started on the selective serotonin reuptake inhibitor sertraline.  At his 2-week follow-up, the patient reports that his mood is "about the same."  He continues to feel sad and unmotivated most days and is short-tempered with his wife and kids.  He has little interest in food, sleeps poorly, and struggles to stay focused at work.  In the first 2-3 days of taking the medication, the patient experienced some mild nausea and anxiety that has since resolved.  He is now tolerating the medication without difficulty but is considering stopping it because he is discouraged by the lack of clear improvement.  Which of the following is the most likely explanation for this patient's lack of response?

A)Comorbid anxiety disorder
B)Development of tolerance
C)Inadequate duration of treatment
D)Medication side effect
E)Poor treatment adherence
F)Treatment-resistant depression
Question
An 8-year-old boy is brought to the office due to severe behavioral problems.  The patient has been suspended from school on two occasions over the past year due to running around in the classroom and talking back to his teachers.  His parents report that he is "in constant motion" and are concerned about his poor grades and inability to follow directions or household routines.  The patient has a history of mild asthma and no other medical problems.  There is a family history of schizophrenia in a maternal uncle.  Physical examination shows marked fidgeting and difficulty staying seated but is otherwise normal.  The boy frequently interrupts his mother while she speaks with the physician.  Treatment with methylphenidate is recommended.  The parents should be educated about which of the following regarding this medication?

A)Acute dystonia
B)Decreased appetite and weight loss
C)Delayed onset of action
D)Polyuria and polydipsia
E)Increased appetite and weight gain
F)Initial worsening of symptoms
G)Life-threatening rash
H)Sedation
Question
A 61-year old woman comes to the office due to insomnia.  She has had difficulty falling asleep since her divorce was finalized 2 months ago and she relocated to a new state to be closer to her daughter and grandchildren.  The patient's sleep is restless and she is frequently awakened by household noises.  During this time, she has also felt anxious and tense.  The patient is overwhelmed by financial worries and the responsibilities of living on her own.  Although previously outgoing, she now avoids opportunities to meet new people and socializes only with her daughter.  The patient has no psychiatric history but does have a history of hypothyroidism treated with levothyroxine.  Physical examination is unremarkable.  TSH is 1.6 µU/mL.  Which of the following is the most likely diagnosis?

A)Acute stress disorder
B)Adjustment disorder
C)Anxiety disorder due to a medical condition
D)Generalized anxiety disorder
E)Medication-induced anxiety disorder
F)Normal stress response
Question
A 35-year-old man comes to the office due to a lack of sexual interest.  He says, "I started seeing this woman 9 months ago and am really attracted to her.  We get along well and things are great, but I just don't want to have sex with her anymore."  On further questioning, the patient reluctantly admits that whenever they have intercourse he has an orgasm in less than a minute and finds this very embarrassing.  He does not have the same problem when he masturbates.  He says, "I'm worried that my girlfriend will leave me, and it's really affecting my self-esteem."  The patient has no other concerns.  His other medical conditions include type 1 diabetes mellitus, chronic insomnia, and a history of major depression.  His medications include insulin glargine, short-acting insulin, and trazodone.  He does not use illicit substances.  Vital signs are within normal limits.  Laboratory results are significant for hemoglobin A1c of 7% and fasting glucose of 130 mg/dL.  Which of the following is the most likely diagnosis?

A)Erectile disorder
B)Male hypoactive sexual desire disorder
C)Premature ejaculation
D)Sexual dysfunction due to diabetes
E)Sexual dysfunction due to major depressive disorder
F)Substance/medication-induced sexual dysfunction
Question
An 82-year-old man is brought to the office by his daughter due to behavioral changes.  She reports that over the past year her father has become increasingly paranoid and frequently talks out loud as if in conversation when no one else is present.  The daughter says, "During a recent argument, he accused me of being an imposter and stealing his personal items.  Also, he used to be very clean, but his personal hygiene has gotten really bad.  He also stopped watching baseball, which used to be his favorite hobby."  The patient's medical history is significant for hypertension, hypercholesterolemia, and type 2 diabetes mellitus.  Medications include lisinopril, simvastatin, and metformin.  The patient has been unable to live independently for the past 2 years due to increasing forgetfulness and an inability to perform self-care.  Vital signs are stable, and physical examination and laboratory evaluation are unremarkable.  Which of the following is the most likely explanation for this patient's behavior?

A)Alzheimer disease
B)Delusional disorder
C)Major depression with psychotic features
D)Schizophrenia
E)Medication-induced psychotic disorder
Question
A 23-year-old woman comes to the emergency department due to right wrist pain after a fall.  The patient fell onto her outstretched hand while walking across the floor at a gym where she exercises regularly.  She has no significant medical history and takes no medications.  The patient says she has tried several diets to lose weight and is currently consuming a vegan diet.  Her last menstrual period was 3 months ago.  She does not use tobacco, alcohol, or illicit drugs.  Temperature is 36.1 C (97 F), blood pressure is 90/58 mm Hg, and pulse is 50/min.  BMI is 18.3 kg/m2.  Weight is 50 kg (110.2 lb) and height is 165 cm (5 ft 5 in).  Physical examination shows tenderness and swelling over the distal radius.  The parotid glands are enlarged and there are fine, soft hairs on her extremities.  Urine pregnancy test is negative.  X-rays reveal a nondisplaced fracture of the right distal radius and generalized radiolucency of the bone.  Which of the following is the most likely diagnosis?

A)Anorexia nervosa
B)Avoidant/restrictive food intake disorder
C)Bulimia nervosa
D)Pellagra
E)Rickets
F)Scurvy
Question
A 78-year-old man comes to the office for a regularly scheduled review of his chronic medical problems.  The patient has hypertension, coronary artery disease, and type 2 diabetes mellitus, for which he takes metformin, atorvastatin, lisinopril, and nitroglycerin as needed for chest pain.  He takes his medications as prescribed and follows diet and exercise instructions.  During the discussion, he hesitates, laughs nervously, and says, "I can't get an erection anymore, and my wife says I have to ask you about getting the 'blue pill.'"  Which of the following is the most appropriate response to this patient's concern?

A)"I can see that you feel uncomfortable talking about this.  It can be a sensitive subject for some men."
B)"I can understand your concern, but at your age, we hesitate to start too many medications."
C)"Medications for erectile dysfunction have significant side effects.  I would not pursue them unless you feel it is important."
D)"This is a very common problem for men as they age.  It is good that you mentioned it."
E)"We can try medication for erectile dysfunction, but it may not be effective at your age."
Question
An 11-year-old girl is brought to the office due to disruptive behavior at home and at school.  Her parents report that she gets in trouble for talking during class and not following instructions.  Although the patient is of above-average intelligence, her grades are poor.  Teachers note that she makes careless mistakes while rushing through tests and frequently forgets to hand in assignments.  At home, she is easily distracted while trying to focus on her homework.  The patient has frequent conflicts with her mother, who says, "Getting her ready for school in the morning is impossible.  We're frequently late because she always misplaces her cell phone and books."  The patient has no other medical history, and developmental milestones are within normal range.  Physical examination shows no abnormalities.  The girl and her parents are willing to consider medication if it will help.  A drug with which of the following mechanisms of action is most appropriate for this patient?

A)Antagonism of alpha-2 adrenergic receptors
B)Antagonism of dopamine D2 receptors
C)Increased availability of norepinephrine and dopamine
D)Increased availability of serotonin
E)Increased availability of serotonin and norepinephrine
F)Positive allosteric modulation of GABA
Question
A 40-year-old woman comes to the office due to worsening anxiety and insomnia over the past 3 months.  She says, "I'm really worried that something is wrong with me.  I was never a particularly anxious person, but now I feel anxious all the time.  Sometimes I feel panicky for no reason; my heart races and I break out in a sweat.  The only benefit is that I have lost 5 pounds (2.3 kg) without even trying."  The patient has no significant medical or psychiatric history.  She drinks 2-3 glasses of wine per week and does not smoke or use illicit drugs.  Blood pressure is 130/90 mm Hg and pulse is 112/min.  On physical examination, the patient is restless and has warm, moist skin and mild hand tremor bilaterally.  Mental status examination is notable for a frightened stare, anxious mood, and rapid speech.  Which of the following is the most likely diagnosis?

A)Alcohol withdrawal
B)Anxiety caused by a medical condition
C)Generalized anxiety disorder
D)Illness anxiety disorder
E)Panic disorder
F)Somatic symptom disorder
Question
A 26-year-old woman comes to the emergency department to report that her life is in danger.  The patient believes that transmitters inserted in her bedroom walls are broadcasting her thoughts to a central government agency that is now plotting to kill her.  On mental status examination, she is fearful and paces the examining room.  The patient makes poor eye contact with the examiner, and her responses are interrupted by frequent pauses during which she appears to talk back to a person who is not present.  She has no medical history.  The patient smokes a pack of cigarettes daily and does not use illicit drugs.  Physical examination shows a thin, disheveled young woman but no abnormalities.  She is hospitalized, improves with medication over the course of 2 weeks, and is discharged on olanzapine.  At this patient's 3-month checkup, which of the following studies should be obtained?

A)Blood urea nitrogen and creatinine
B)Complete blood count
C)Electrocardiogram
D)Fasting glucose and lipid panel
E)Prolactin level
F)Thyroid function tests
Question
A 16-year-old woman comes to the office due to malodorous vaginal discharge.  The patient arrives with her mother, who stays in the examination room for the evaluation.  The patient has had increased vaginal discharge for the past 2 days but no abnormal vaginal bleeding or abdominal or pelvic pain.  She is sexually active with a new partner and uses a progestin-releasing subdermal implant for contraception.  Her last menstrual period was 2 weeks ago.  She has no chronic medical conditions and takes no medications.  Vital signs are normal.  The patient appears anxious.  Abdominal examination shows no tenderness or palpable masses.  When the pelvic examination is attempted, the patient says that she is "embarrassed and anxious" and refuses the examination.  Which of the following is the most appropriate course of action?

A)Administer a benzodiazepine and proceed with the examination
B)Ask the patient's mother to leave the room and proceed with the examination
C)Distract the patient with questions while performing the examination
D)Do not perform the examination and ask the patient if she will perform self-collection for testing
E)Obtain permission from the patient's mother to proceed with the examination
Question
A 32-year-old woman comes to the office due to long-standing anxiety.  She reports being excessively worried about everyday events and constantly feeling tense and unable to relax.  The patient is diagnosed with generalized anxiety disorder and treated with a selective serotonin reuptake inhibitor.  She returns to the office for a 2-month follow-up and reports that her overall anxiety has improved, but she continues to have periods of increased anxiety before bedtime that keep her awake at night.  The patient works as a 911 emergency dispatch operator and would be willing to add a medication at bedtime but needs to be "clearheaded" for work.  Which of the following medications is most appropriate for this patient's condition?

A)Chlordiazepoxide
B)Diazepam
C)Flurazepam
D)Lorazepam
E)Quetiapine
Question
A 9-year-old boy is brought to the pediatrician due to poor school performance and difficulty making friends.  His parents say he always had "very high energy," but they noticed that this got worse after they moved 8 months ago, which they attributed to a normal adjustment period.  Over the past several months, however, the parents have received repeated phone calls from his teachers, reporting that he does not listen in class, has difficulty staying in his seat, turns in assignments late without his name, and frequently disrupts the class by talking out of turn.  At home the boy forgets to do his daily chores, regularly loses his books and homework, and delays the family by taking too long getting ready for school in the morning.  When his older brother teases him about this, the patient becomes irritable, and on a few occasions has shouted profanities and tried to hit his brother.  Which of the following is the most likely explanation for this patient's behavior?

A)Adjustment disorder
B)Attention-deficit hyperactivity disorder
C)Conduct disorder
D)Social anxiety disorder
E)Oppositional defiant disorder
Question
A 22-year-old man comes to the emergency department due to sudden-onset pain and stiffness on one side of his neck.  He was diagnosed with a psychiatric disorder 2 days ago and has been adherent with his newly prescribed medication.  For the past few months, the patient has heard voices telling him to quit his job, and he believes that his girlfriend's parents are monitoring his sexual thoughts with a special device.  He has been socially withdrawn and has difficulty showering regularly.  Vital signs are within normal limits.  The head is tilted to one side and cannot be straightened without considerable pain.  The most likely cause of this patient's symptoms is antagonism of which of the following receptors?

A)Alpha-1 adrenergic
B)Dopaminergic D2
C)GABA-A
D)Histaminergic H1
E)Muscarinic M1
F)Serotonergic 5-HT2A
Question
A 20-year-old college student comes to the office due to fatigue and decreased exercise tolerance.  She says, "I used to run 5 miles every day, but for the past month I get out of breath after running 3 miles.  I need to keep running so I don't get any fatter than I already am."  The patient also reports difficulty concentrating in class, depressed mood, constipation, and abdominal bloating.  Her appetite is normal, but she admits to occasionally forcing herself to vomit to prevent weight gain.  Physical examination is significant for dry skin, painless bilateral parotid swelling, pharyngeal erythema, and hypoactive bowel sounds.  Temperature is 36.2 C (97.2 F), blood pressure is 100/60 mm Hg, pulse is 62/min, and respirations are 16/min.  BMI is 17.5 kg/m2.  Which of the following is the most likely diagnosis?

A)Anorexia nervosa
B)Body dysmorphic disorder
C)Bulimia nervosa
D)Hypothyroidism
E)Irritable bowel syndrome
F)Major depressive disorder
Question
A 46-year-old woman comes to the office due to sexual difficulties for the past year.  The patient rarely thinks about sex anymore and frequently declines sex when her husband tries to initiate.  She is attracted to her husband and worries that he assumes she is losing interest in him.  Although vaginal lubrication is adequate during sex and the patient can reach orgasm, she finds that her mind wanders during the experience.  She is unsure what is causing a decrease in her sex drive, although she notes that she received a promotion 3 months ago and has been working longer hours since then.  The patient describes her marriage as stable and loving.  She has a history of a depressive episode and currently takes bupropion.  Physical examination and laboratory studies, including thyroid function tests, show no abnormalities.  Which of the following is the most likely diagnosis?

A)Adjustment disorder
B)Antidepressant-induced sexual dysfunction
C)Female orgasmic disorder
D)Female sexual interest/arousal disorder
E)Genito-pelvic pain/penetration disorder
F)Normal menopausal changes
Question
A 52-year-old woman comes to the office for a checkup.  Her medical problems include hypertension and hypercholesterolemia.  The patient has a long-standing relationship with her primary care physician, who has treated her children as well.  During the examination, she confides that she has been "down" since her youngest child left for college 2 months ago.  The patient is worried about her daughter being away from home for the first time and whether she will be successful at school.  At work, the patient occasionally has lapses of concentration when worrying about whether her daughter is okay, but it has not affected her productivity.  She says, "I still enjoy going out with my husband but all we do is talk about our kids."  The patient has occasional insomnia and tension headaches but notes that these are nothing new.  Physical examination is normal.  Which of the following is the most likely explanation for this patient's condition?

A)Adjustment disorder with depressed mood
B)Generalized anxiety disorder
C)Major depressive disorder
D)Normal sadness
E)Persistent depressive disorder
Question
A 60-year-old woman comes to the office due to uncontrollable movements that have worsened.  Medical history is significant for hypertension, hyperlipidemia, osteoarthritis, and schizophrenia, and she has been on a stable medication regimen for years.  Throughout the interview, she exhibits facial grimacing, lip smacking, and twisting movements of her hands and feet.  The patient has lived in a group home for many years and worries that her housemates are avoiding her because her symptoms make her look "strange."  On mental status examination, the patient makes poor eye contact and is generally distrustful but has no specific delusions or hallucinations.  Which of the following is the most likely diagnosis?

A)Acute dystonia
B)Acute psychotic episode
C)Akathisia
D)Neuroleptic-induced parkinsonism
E)Neuroleptic malignant syndrome
F)Tardive dyskinesia
G)Tic disorder
Question
A 63-year-old man comes to the office for follow-up of hypertension.  At his last appointment, he revealed that he had increased his intake of 12-oz cans of beer from 3 to 6 cans daily due to stress at work.  The physician advised him to seek help for his alcohol use because it could be causing his elevated blood pressure and has many negative health risks.  The patient now tells the physician, "I thought about what you said.  I know my alcohol use has gotten out of hand and is affecting my health.  My wife and daughter also say that I need to quit.  I have made an appointment with a counselor to talk about my options."  Which of the following best describes this patient's stage of behavioral change?

A)Action
B)Contemplation
C)Intellectualization
D)Precontemplation
E)Preparation
F)Rationalization
Question
A 12-year-old boy is brought to the office due to behavioral problems.  His parents are upset about his poor grades and recent school suspension for setting fires in the locker room and stealing another student's cell phone.  They say that the boy has always been impulsive, had a lot of energy, and become angry and argumentative easily, but over the past year his aggressive behavior "has gotten out of control."  The patient says that the fires were an "accident," although he admits to being angry at the coach for kicking him off the basketball team.  Regarding the theft, he jokingly says, "People deserve to have their phones stolen when they are stupid enough to leave them in plain sight."  The patient has a history of getting into trouble for talking back to teachers, skipping class, and getting into fights since the fourth grade.  He has no medical history.  There is a family history of bipolar disorder and alcohol dependence.  The patient smokes cigarettes occasionally but denies illicit drug or alcohol use.  Which of the following is the most likely diagnosis in this patient?

A)Antisocial personality disorder
B)Attention-deficit hyperactivity disorder
C)Conduct disorder
D)Intermittent explosive disorder
E)Oppositional defiant disorder
F)Pyromania
Question
A 28-year-old man is brought to the emergency department by his roommate, who is concerned about his change in behavior over the past 2 weeks.  The roommate describes the patient as "a regular guy who is usually very responsible."  Last week, the patient abruptly quit his job as a computer programmer and started placing large bets on an online gambling site because he was "sure to make millions."  The roommate says that the patient has been staying up most nights scribbling notes for his autobiography on small scraps of paper.  The patient says, "My new mission is to spread understanding."  He denies any alcohol or drug use, which his roommate affirms.  This patient is most likely to exhibit which of the following additional findings?

A)Flat affect
B)Low self-esteem
C)Poor hygiene
D)Pressured speech
E)Psychomotor slowing
F)Social withdrawal
G)Thought blocking
Question
A 25-year-old graduate student with a history of recurrent ear infections as a child feels anxious and sweats when she is in the examination room with her primary care physician.  She recently got a roommate, a nursing student, who leaves her stethoscope on the coffee table after returning from class.  The patient sweats and feels her heart start to race whenever she sees the stethoscope.  This patient's response to her roommate's stethoscope is an example of which of the following phenomena?

A)Classical conditioning
B)Negative punishment
C)Negative reinforcement
D)Operant conditioning
E)Reaction formation
F)Regression
Question
A 13-year-old boy is brought to the office due to anxiety and behavioral problems at school.  Although he has good grades, he frequently feels overwhelmed at school and his mother worries that he is socially awkward and has no friends.  The patient often appears anxious and uncomfortable in new social situations.  He insists on sitting in the same row and seat in every class and has had several inappropriate outbursts when told by teachers that this was not possible.  After school, the patient prefers to spend time at home.  His mother observes that he is obsessed with dates and comparing solar and lunar calendars and can discuss this for hours, but has difficulty shifting to other topics of conversation.  The patient was born at full term, and developmental milestones were within normal range.  Physical examination is unremarkable.  Which of the following is the most likely explanation of this patient's behavior?

A)Autism spectrum disorder
B)Obsessive compulsive disorder
C)Obsessive compulsive personality disorder
D)Schizoid personality disorder
E)Separation anxiety disorder
F)Social anxiety disorder
Question
A 13-year-old girl is brought to the office for a routine visit.  Although the patient is doing well in school and has friends, her mother is concerned about a change in her behavior over the past several months.  They had previously enjoyed spending time together, but now the patient is frequently moody and spends most of her time on the computer in her locked bedroom.  The mother also observes that her daughter gets upset for no reason and has used swear words on occasion.  When the mother recently complimented her party outfit, the girl screamed, "I look hideous and I'm not going," and stormed out of the room.  Later that day, she seemed fine and went to the party.  On examination, the patient is pleasant and cooperative but makes little eye contact and asks if she can keep her tank top on during the examination.  Which of the following is the most likely explanation for this girl's behavior?

A)Body dysmorphic disorder
B)Borderline personality disorder
C)Cyclothymic disorder
D)Disruptive mood dysregulation disorder
E)Normal adolescent behavior
F)Oppositional defiant disorder
Question
A 2-year-old boy is brought to the clinic by his mother, who is concerned about his language development.  She says, "I know that children speak at different ages and he is still young, but I am concerned that his speech isn't progressing and that he needs to interact with other children more.  His day care teacher says that he is shy and sits in a corner playing with his favorite truck.  We have also been busy with a recent move and have not given him as much attention, which doesn't help."  The patient's birth history, medical history, and audiology screening are unremarkable.  His mother is 6 months pregnant with her second child, and the family moved to a new home 3 months ago.  In the examination room, the patient sits in a corner, avoiding eye contact with the doctor and repeatedly spinning the wheels of a toy truck.  He does not respond to his name.  He makes occasional grunting sounds but says no fully formed words.  When his mother tries to take his truck away, he begins screaming and pulls it back.  Which of the following is the most likely diagnosis in this patient?

A)Adjustment disorder
B)Autism spectrum disorder
C)Language disorder
D)Normal variation in development
E)Selective mutism
F)Social anxiety disorder
G)Social (pragmatic) communication disorder
Question
A 30-year-old man comes to the office due to depression.  He describes a 4-week history of severely depressed mood, loss of motivation, loss of interest, and sleeping 12 hours a day.  The patient has difficulty concentrating and worries that it is interfering with his work.  He is pessimistic about the future but has no suicidal thoughts.  The patient had a previous depressive episode in his 20s that responded rapidly to sertraline.  He felt "really good and energetic" after a few days of treatment but discontinued the antidepressant a week later as he did not feel that he needed it.  He drinks 1 or 2 beers on social occasions.  Physical examination is normal.  Which of the following would be most important to assess prior to initiating pharmacologic treatment for this patient?

A)Complete blood count
B)Comprehensive metabolic panel
C)Electrocardiogram
D)History of mania
E)History of sexual dysfunction
F)Urine toxicology
Question
A 31-year-old man is brought to the emergency department by the police after he assaulted a stranger who he thought was following him.  The police report indicates that when the patient was arrested, he was shouting, "You don't understand, I'm the one who needs protection-the Russians are after my secrets!"  In the emergency department, the patient is diaphoretic and tremulous.  He is very easily distracted and cannot give a clear history.  Temperature is 37 C (98.6 F), blood pressure is 164/102 mm Hg, and pulse is 112/min with a normal rhythm.  Extraocular movements are intact, and the pupils are dilated.  His speech is loud, rapid, and difficult to interrupt.  Which of the following is the most likely diagnosis?

A)Anticholinergic toxicity
B)Bipolar disorder, manic episode
C)Cocaine intoxication
D)Delusional disorder, persecutory type
E)Opioid withdrawal
F)Phencyclidine intoxication
G)Schizophreniform disorder
Question
A 35-year-old man comes to the office due to problems with irritability, anxiety, and low self-esteem.  He was recently fired due to poor work performance at his data entry job and worries about finding a new job and supporting himself.  The patient is currently in a relationship but mentions that his girlfriend is upset by his frequent lateness and forgetfulness and has threatened to leave him.  Further history indicates that his problems are long-standing and date back to childhood when he had behavioral problems in school.  The patient frequently feels overwhelmed due to being disorganized.  He frequently procrastinates, is bored easily at work, and jumps to another project before completing what he is working on.  The patient drinks 3 or 4 beers a week and smokes marijuana twice a month.  On mental status examination, he is cooperative and talkative but appears tense, restless, and easily distracted.  Which of the following is the most likely diagnosis in this patient?

A)Adjustment disorder
B)Attention-deficit hyperactivity disorder
C)Bipolar disorder
D)Borderline personality disorder
E)Generalized anxiety disorder
F)Persistent depressive disorder
G)Substance-induced mood disorder
Question
A 27-year-old woman comes to the office due to concerns about her weight.  She is frustrated about gaining a few pounds, although she maintains a well-balanced diet and exercises for an hour 3 times a week.  The patient has been worried for years that her thighs are "huge" and "covered in cellulite."  She always wears loose-fitting pants and spends hours a day researching leg exercises and measuring the circumference of her thighs.  The patient generally stays at home and avoids social activities as she feels people tend to stare at her.  She weighs 58 kg (127.9 lb) and is 170 cm (5 ft 7 in) tall.  Physical examination is normal.  Which of the following is the most likely diagnosis?

A)Agoraphobia
B)Anorexia nervosa
C)Avoidant personality disorder
D)Body dysmorphic disorder
E)Bulimia nervosa
F)Delusional disorder
G)Obsessive-compulsive disorder
H)Social anxiety disorder
Question
An 18-year-old boy is brought to the emergency department by his mother due to his strange behavior.  The mother says that he laughs inappropriately and seems sluggish.  The boy has recently been hanging out with a new group of friends that he describes as "really cool."  He has also stopped attending family events and appears apathetic about his schoolwork.  When asked in private, the boy admits smoking marijuana.  Which of the following physiologic effects is most likely to be observed in this patient?

A)Bradycardia
B)Conjunctival injection
C)Miosis
D)Mydriasis
E)Nystagmus
F)Respiratory depression
Question
An 18-year-old woman comes to the office due to low energy, decreased appetite, insomnia, poor concentration, and decreased interest in her daily activities.  She has lost 3.6 kg (8 lb) over the last month.  The patient has no psychiatric history and has no current or significant medical illnesses.  Physical examination and laboratory evaluation are unremarkable.  Treatment with medication is initiated.  Three weeks later, the patient is brought to the emergency department by her mother for not sleeping at all for 3 nights.  The patient reports that she does not feel tired despite lack of sleep.  She spent a large amount of money on plane tickets and plans to travel around the world to promote world peace.  Her mother adds that she herself has suffered from severe mood swings in the past.  This patient was most likely started on which of the following medications 3 weeks ago?

A)Carbamazepine
B)Lamotrigine
C)Lithium
D)Olanzapine
E)Sertraline
F)Valproate
Question
A 31-year-old woman comes to the office for a yearly checkup.  The patient has no medical conditions but expresses dissatisfaction with her appearance and wishes she could lose weight more easily.  She reports occasional constipation and fatigue.  The patient exercises daily and drinks 1 or 2 glasses of wine 2-3 times a week when socializing with friends.  Weight is 58.1 kg (128 lb) and height is 157.5 cm (5 ft 2 in).  BMI is 23.4 kg/m2.  Routine laboratory evaluation shows a potassium level of 3.1 mEq/L.  Physical examination is most likely to show which of the following abnormalities?

A)Bradycardia
B)Diaphoresis
C)Goiter
D)Hypertension
E)Lanugo
F)Malar rash
G)Parotid gland enlargement
Question
A 21-year-old woman comes to the office due to constipation and vague abdominal pain.  She says, "For the last few months I have felt bloated and my stomach doesn't feel right."  Review of systems is notable for irregular menses and occasional fatigue.  Medical history is significant for a broken femur at age 6 but is otherwise noncontributory.  Family history is significant for Graves disease in her father and primary myelofibrosis in her paternal grandmother.  The patient smokes a pack of cigarettes daily and drinks wine when relaxing with friends.  She does not use illicit drugs.  Temperature is 37.2 C (99 F), blood pressure is 90/60 mm Hg, pulse is 118/min, and respirations are 16/min.  BMI is 25.6 kg/m2.  Physical examination shows eroded enamel of the teeth.Laboratory results are as follows: <strong>A 21-year-old woman comes to the office due to constipation and vague abdominal pain.  She says, For the last few months I have felt bloated and my stomach doesn't feel right.  Review of systems is notable for irregular menses and occasional fatigue.  Medical history is significant for a broken femur at age 6 but is otherwise noncontributory.  Family history is significant for Graves disease in her father and primary myelofibrosis in her paternal grandmother.  The patient smokes a pack of cigarettes daily and drinks wine when relaxing with friends.  She does not use illicit drugs.  Temperature is 37.2 C (99 F), blood pressure is 90/60 mm Hg, pulse is 118/min, and respirations are 16/min.  BMI is 25.6 kg/m<sup>2</sup>.  Physical examination shows eroded enamel of the teeth.Laboratory results are as follows:   Which of the following is the most likely cause of this patient's condition?</strong> A)Acute pancreatitis B)Alcohol use disorder C)Bulimia nervosa D)Hypothyroidism E)Irritable bowel syndrome F)Sjögren syndrome <div style=padding-top: 35px> Which of the following is the most likely cause of this patient's condition?

A)Acute pancreatitis
B)Alcohol use disorder
C)Bulimia nervosa
D)Hypothyroidism
E)Irritable bowel syndrome
F)Sjögren syndrome
Question
A 29-year-old woman comes to the office due to depression.  Since breaking up with her boyfriend last month, she has been extremely sad and has difficulty getting out of bed.  She describes sleeping 16 hours a day, increased appetite, a 4.5-kg (10-lb) weight gain, low energy, decreased concentration, and loss of interest in socializing with her friends and family.  The patient had 2 similar episodes at age 23 and 27.  She also describes brief periods in the past, lasting several days, when she was uncharacteristically confident and optimistic, successfully juggled 3 part-time jobs, and felt well rested and energetic despite sleeping only 3-4 hours a night.  The patient drinks a glass of wine several times a week but does not use tobacco or illicit drugs.  Which of the following is the most likely diagnosis?

A)Adjustment disorder with depressed mood
B)Bipolar I disorder
C)Bipolar II disorder
D)Cyclothymic disorder
E)Recurrent major depressive disorder
Question
A healthy 8-year-old boy is brought to the office by his parents for an annual checkup.  Both parents express concern about their son's behavior.  The mother says, "He never listens and I am always worried that he is going to run out into the street without looking.  He rarely sits still and is always running and jumping on the furniture."  The father adds, "He talks all the time, interrupts me when I'm speaking, and, despite multiple reminders, forgets to do his chores and misplaces or loses his books or sporting equipment.  We thought he would grow out of it as he got older, but it seems to be getting worse."  Physical examination is unremarkable.  Which of the following would be most helpful in establishing the diagnosis?

A)Brain imaging
B)Hearing test
C)Quantitative EEG
D)Speech and language evaluation
E)Teacher evaluations
F)Toxicology screening
Question
A 27-year-old man is brought to the emergency department by his wife.  She says that he has been "acting crazy" for the last 2 weeks.  He has hardly slept for the past 7 days and instead has worked on miscellaneous projects around the house.  The patient spent several thousand dollars on new power tools to accomplish these tasks.  When questioned, his speech is rapid and frenzied.  He feels "spectacular" and is creating an "architectural masterpiece."  The patient has had 2 previous depressive episodes.  Which of the following medications is the most appropriate agent for long-term management of this patient?

A)Bupropion
B)Chlorpromazine
C)Haloperidol
D)Lorazepam
E)Paroxetine
F)Valproate
Question
A 28-year-old man is hospitalized due to the acute onset of severe depression and suicidal ideation for several days.  He was recently thrown out of his mother's home after stealing her money to support his drug habit.  The patient reports fatigue and vivid, disturbing dreams.  The nursing staff note that he is irritable, withdrawn, hypersomnolent, and hyperphagic.  The patient has an extensive history of substance abuse and has been hospitalized previously for alcohol detoxification.  Blood pressure is 110/80 mm Hg and pulse is 64/min.  Physical examination shows old injuries from a motorcycle accident but no other abnormalities.  On mental status examination, he appears lethargic and dysphoric.  This patient's current symptoms are most likely the result of which of the following conditions?

A)Alcohol withdrawal
B)Benzodiazepine withdrawal
C)Cannabis withdrawal
D)Cocaine withdrawal
E)Major depressive disorder
F)Opiate withdrawal
Question
A 26-year-old woman comes to the office due to recent weight gain.  She has eaten more than usual over the last 5 months, has gained 3.2 kg (7 lb), and feels guilty and depressed about it.  Further questioning reveals that she consumes a large pizza and two large bags of chips in one sitting several times a week.  Afterward, the patient feels ashamed about being unable to control her intake and fasts to make up for it.  She is very distressed about being unable to lose weight despite exercising 2-3 hours a day.  Vital signs are within normal limits.  BMI is 23.7 kg/m2.  Despite being told that her BMI is normal, the patient insists that she is overweight.  Which of the following is the most likely diagnosis?

A)Adjustment disorder with depressed mood
B)Anorexia nervosa
C)Binge-eating disorder
D)Body dysmorphic disorder
E)Bulimia nervosa
F)Normal behavior
Question
A 26-year-old man is brought to the emergency department by police after assaulting customers in a restaurant.  He explains that he meant no harm and was just trying to talk to people and touch them to "heal their pain."  The patient has been up all night for the past 10 days devising a global strategy to end world hunger and has written a hundred-page manifesto documenting his ideas.  Over this same period, he began hearing a voice telling him that he is "God's true son" and will need to sacrifice his life.  The patient has no psychiatric or medical history.  He drinks alcohol socially but does not use illicit drugs.  Physical examination is normal.  On mental status examination, the patient paces continuously.  His mood is very irritable, and his speech is loud, rapid, and difficult to interrupt.  Which of the following is the most likely diagnosis in this patient?

A)Bipolar I disorder with psychotic features
B)Bipolar II disorder
C)Brief psychotic disorder
D)Delusional disorder
E)Schizophreniform disorder
F)Substance-induced psychotic disorder
Question
A 79-year-old man is brought to the office by his wife for evaluation of memory impairment.  The patient is a recently retired professor who began having memory problems a month ago.  He has been forgetting to take his medication, has had difficulty remembering the names of his grandchildren, and has neglected to turn off the stove on more than one occasion.  The patient used to enjoy reading and playing with his grandchildren but has recently stopped doing both of these activities.  He also has difficulty staying asleep, low appetite, and decreased energy.  The patient has hypertension and type 2 diabetes, for which he takes medications.  He has a family history of Alzheimer disease.  The patient says, "I just feel so worthless since retiring."  Temperature is 36.7 C (98.1 F), blood pressure is 119/78 mm Hg, pulse is 74/min, and respirations are 14/min.  Neurological examination reveals no focal deficits.  Montreal Cognitive Assessment score is 23 (normal: ≥26/30).  Which of the following is the most likely diagnosis?

A)Alzheimer disease
B)Dementia with Lewy bodies
C)Depression-related cognitive impairment
D)Frontotemporal dementia
E)Normal aging
F)Vascular dementia
Question
A 28-year-old man comes to the office due to persistent fatigue.  For the past several weeks, he has been having trouble sleeping and is frequently late for work as he has difficulty getting out of bed.  At work, his mind wanders, and his supervisor has commented that the quality of his work has deteriorated.  The patient says, "I've been feeling down and don't feel like doing anything.  My appetite is poor.  I no longer go out with my friends and haven't gone to the gym in weeks."  The patient has no significant medical or psychiatric history.  He has 1 or 2 alcoholic drinks several days a week but does not use illicit drugs.  Routine laboratory evaluation, including thyroid function tests and serum B12 level, is normal.  Physical examination is unremarkable.  Mental status examination shows a cooperative man with sad affect.  No delusions are elicited.  The patient says he sometimes wishes he could "go to sleep and not wake up," but has no suicidal intent or plan.  First-line pharmacotherapy for this patient most likely involves a drug with which of the following mechanisms of action?

A)Antagonism of dopamine D2 receptors
B)Antagonism of serotonin 5-HT2 receptors
C)Inhibition of sodium channels
D)Inhibition of the serotonin transporter
E)Inhibition of monoamine oxidase
F)Potentiation of the effects of endogenous GABA
Question
A 42-year-old man comes to the office for a checkup.  He has no medical symptoms but says, "I'm just feeling stressed and not like myself."  The patient is going through a contentious divorce.  He is outraged that his wife is pursuing full custody of their children and says, "I can't believe she is doing this to me."  His job as a legal researcher had been a welcome distraction from his upsetting personal life until yesterday when he was called to his supervisor's office for yelling at the receptionist when the copy machine ran out of ink.  Which of the following is the best explanation for this patient's behavior toward the receptionist?

A)Acting out
B)Denial
C)Displacement
D)Intellectualization
E)Reaction formation
F)Splitting
G)Sublimation
Question
A 40-year-old woman is brought to the emergency department by her roommate due to significant left leg weakness.  The symptom began 3 days ago after her father had a heart attack.  There is no personal or family history of neurological disease; surgical history includes liposuction of the thighs and varicose vein removal.  She does not use tobacco, alcohol, or illicit drugs.  Temperature is 36.7 C (98.1 F), blood pressure is 123/81 mm Hg, pulse is 62/min, and respirations are 14/min.  Physical examination reveals symmetric 2+ deep tendon reflexes as well as normal muscle bulk and tone bilaterally in the upper and lower extremities.  Laboratory testing and neurologic imaging reveal no abnormalities.  Which of the following is the most likely diagnosis?

A)Body dysmorphic disorder
B)Conversion disorder
C)Factitious disorder
D)Illness anxiety disorder
E)Malingering
F)Somatic symptom disorder
Question
A 72-year-old man is hospitalized due to a leg injury following a high-speed motor vehicle collision.  Medical history is significant for hypertension, hypercholesterolemia, and hearing loss.  The patient drinks a glass of wine every day.  Examination shows localized swelling and severe tenderness under the right knee; the presence of a tibial fracture is confirmed by imaging.  The patient undergoes open reduction and internal fixation without any perioperative complications.  On the first postoperative night, he describes visual hallucinations of his deceased brother in the hospital room with him and perseverates about people invading his home.  Vital signs are normal.  The patient is disoriented and unable to cooperate with the mental status examination.  He appears to respond to internal stimuli.  Which of the following interventions is most likely indicated to prevent further worsening of his condition?

A)Facilitating uninterrupted nocturnal sleep
B)Providing sedation by administering a benzodiazepine
C)Reducing daytime visual stimulation by closing window blinds
D)Reducing hyperstimulation by removing hearing aids
E)Treating pain with nonpharmacologic aids only
Question
A 14-year-old boy is brought to the office by his parents, who are worried about his reaction to their recent decision to divorce.  Despite their efforts to be supportive and engage him, he has not expressed any feelings directly about the divorce.  When the patient is evaluated alone, he shares that he feels his parents are angry with him, although he cannot think of any instances when they expressed any anger toward him.  On examination, the patient appears sullen and reports his mood is "fine."  This patient is most likely using which of the following defense mechanisms?

A)Acting out
B)Displacement
C)Identification
D)Projection
E)Reaction formation
F)Regression
Question
A 72-year-old retired man with end-stage renal disease becomes agitated during dialysis and attempts to abruptly leave in the middle of the procedure.  The patient accuses the staff of violating his rights and stealing his belongings but then appears to fall asleep.  Fifteen minutes later, he becomes violent and requires restraints.  Medical history is significant for hypertension, type 2 diabetes mellitus, hyperlipidemia, and peripheral vascular disease.  The patient has a history of major depression that has been in remission for 15 years.  His wife says that he has experienced mild memory and word-finding difficulties over the last year and that he often needs to make lists to remind himself of errands; however, she has never witnessed this type of agitated behavior.  Temperature is 38.3 C (100.9 F), blood pressure is 112/63 mm Hg, pulse is 93/min, and respirations are 18/min.  The patient has a left carotid bruit, and a dialysis catheter is present in the right internal jugular vein.  Bronchial breath sounds are heard at the right lung base.  During neurologic evaluation, he is confused but cooperative with no focal findings.  Which of the following is the most likely explanation for this patient's current behavior?

A)Alzheimer disease
B)Brief psychotic disorder
C)Delirium
D)Depression with psychotic features
E)Frontotemporal dementia
F)Vascular dementia
Question
A 44-year-old man comes to the office due to low mood, impaired concentration, increased sleep and appetite, feelings of heaviness in his arms and legs, and loss of energy.  He is having difficulty at work as he is overly sensitive to criticism.  Although his boss has told him not to worry, the patient is concerned that his job is in jeopardy due to poor performance.  His symptoms started 8 months ago without any clear-cut precipitating event.  The patient received adequate trials of 3 different antidepressants without improvement and has been off of medication for the past several weeks.  His physician is now considering electroconvulsive therapy (ECT).  The patient declines ECT and asks to try another medication.  The physician then considers phenelzine.  The presence of which of the following additional symptoms would make the patient more likely to respond to this medication?

A)Auditory hallucinations
B)Decreased libido
C)Delusions
D)Memory impairment
E)Mood reactivity
F)Rapid speech
Question
A 17-year-old girl is brought to the office by her parents due to recurrent episodes where "she seems to be lost in her own world."  The parents say that the episodes began about 2 years ago, but seem to be occurring more frequently since a difficult breakup with her boyfriend several months ago.  The patient acknowledges that she seems to "space out" easily.  She says, "I often feel numb, like I'm in a fog or a dream.  Sometimes, I feel as if I'm not myself, as if things are happening to someone else."  The patient uses nonsteroidal anti-inflammatory drugs for menstrual cramps.  She has no other medical conditions.  She smokes cigarettes but does not use alcohol or illicit drugs.  Physical and neurologic examinations are normal.  On mental status examination, the patient is cooperative with a mildly anxious mood and flat affect.  Which of the following is the most likely diagnosis?

A)Absence seizures
B)Brief psychotic disorder
C)Depersonalization/derealization disorder
D)Dissociative amnesia
E)Dissociative identity disorder
F)Post-traumatic stress disorder
Question
A 45-year-old woman brings her stepson to the office for a well-child visit.  She never wanted children due to her unhappy childhood and wishes her marriage did not include being a stepmother to her husband's 5-year-old son.  In the waiting room, the woman seems excessively concerned when the boy accidentally trips and is clearly uninjured.  During the visit, she tells the physician that she adores the child and describes her elaborate preparations for his birthday party.  Which of the following defense mechanisms best explains this woman's behavior?

A)Displacement
B)Projection
C)Rationalization
D)Reaction formation
E)Splitting
F)Suppression
Question
A 54-year-old man arrives for an appointment with a new physician.  At the front desk, the receptionist tells him that the physician is running behind schedule and gives him several forms to complete regarding his personal and family medical history.  The patient politely agrees to fill them out and has a seat in the waiting area.  However, when his paperwork is reviewed by the physician, it contains only the patient's signature.  The patient says, "I'm sure a doctor with your education can take a good history."  When asked if anything is wrong he says, "Oh, nothing, I'm fine."  Which of the following best describes this patient's behavior?

A)Acting out
B)Displacement
C)Passive aggression
D)Reaction formation
E)Splitting
F)Suppression
G)Undoing
Question
A 63-year-old woman with a history of metastatic breast cancer comes to the office due to depressed mood.  Over the past month, the patient has become increasingly sad and frequently cries when thinking about her poor prognosis and dying.  She has lost 4.5 kg (10 lb) over the past month.  Her energy level is low, and she has difficulty falling asleep and frequent nighttime awakenings.  The patient has become very withdrawn, doesn't answer the phone, and no longer looks forward to family visits.  She feels bad about not wanting to be around her grandchildren.  On mental status examination, the patient is alert and oriented with depressed mood and affect.  She has no suicidal ideation.  Which of the following symptoms is most indicative of major depressive disorder in this patient?

A)Loss of interest in family
B)Low energy
C)Sleep disturbance
D)Thoughts of dying
E)Weight loss
Question
A 71-year-old woman is brought to the emergency department with suicidal ideation.  Medical history includes major depressive disorder as well as chronic knee and back pain due to injuries sustained in a motor vehicle collision.  Developmental history is significant for physical abuse as a child.  Family history is significant for bipolar disorder and completed suicide in her father.  The patient reports increasing depression since losing her job a year ago and separating from her husband of 40 years 6 months ago.  She has a history of 2 prior hospitalizations for depression and a previous suicide attempt.  The patient is recovering from alcohol use disorder and attends weekly Alcoholics Anonymous meetings at her church.  Although she lives alone, she is close to her daughter, who lives nearby.  The patient reports that she owns a firearm that is kept in a locked cabinet.  Which of the following is the strongest risk factor for completed suicide in this patient?

A)Access to firearms
B)Age >70
C)Female sex
D)History of childhood trauma
E)History of completed suicide in a parent
F)History of suicide attempt
G)Major depressive disorder
H)Recent marital separation
Question
A 48-year-old man begins psychotherapy due to depression and escalating conflicts with his boss.  He makes progress in the therapy sessions and reports feeling less depressed.  The patient has also gained insight into how his father was rarely available to him while growing up and recently told his father, "I am angry that you were never there for me."  The following week, the psychiatrist unexpectedly cancels a session to attend to an emergency.  At the next appointment, the patient says, "I feel as if you are not interested in listening to me, like I am not your priority."  Which of the following is the most likely explanation of this patient's attitude toward his psychiatrist?

A)Acting out
B)Displacement
C)Projection
D)Reaction formation
E)Regression
F)Transference
Question
A 45-year-old man is hospitalized due to suicidal ideation.  Over the past month, the patient has become increasingly depressed, withdrawn, and physically restless.  He has had increasing difficulty concentrating at his job as a computer programmer.  The patient's appetite has been poor, and he has had difficulty falling and staying asleep.  He stopped going to work last week and refused to leave the house until his hospitalization.  The patient has no medical or psychiatric history.  Physical examination is unremarkable apart from a 4.5-kg (10-lb) weight loss.  On mental status examination, the patient appears depressed and anxious.  He reports that he is responsible for "all the evil in the world" and has heard a voice for the past week telling him that he does not deserve to live.  Which of the following is the most likely diagnosis in this patient?

A)Bipolar disorder with psychotic features
B)Delusional disorder
C)Major depressive disorder with psychotic features
D)Schizoaffective disorder
E)Schizophrenia
F)Schizophreniform disorder
Question
A 72-year-old woman is admitted to the hospital due to abnormal vaginal bleeding.  Uterine cancer is diagnosed and the patient is scheduled for a hysterectomy.  Following successful surgery, the patient compliments her surgeon, exclaiming that she is "a brilliant doctor who saved my life."  After an uneventful recovery, the patient is ready to return home.  On the day of discharge, a nurse informs her that the surgeon is running late due to an emergency.  The patient responds angrily that the surgeon is "terrible and doesn't care about patients."  Which of the following is the most likely explanation for this patient's behavior?

A)Acting out
B)Displacement
C)Passive aggression
D)Projection
E)Reaction formation
F)Splitting
Question
A resident physician is finishing up her shift when she receives a page that reads, "Patient in Room 121 asking to leave."  The patient was admitted 2 hours ago for worsening dyspnea.  On entering the room, the resident attempts to engage in a discussion about why the patient is requesting to leave.  In response, the patient pulls out his nasal cannula and breathlessly says, "You don't know anything; you're not even a real doctor!"  The resident is reminded of her father, who has frequently belittled her accomplishments.  She angrily informs the patient, "You can't leave and I'm ordering haloperidol to calm you down."  Which of the following best explains the resident's response to the patient?

A)Countertransference
B)Passive aggression
C)Projection
D)Reaction formation
E)Regression
F)Transference
Question
A medical student just finished the USMLE Step 1 exam and is anxious about her performance.  She is especially unnerved when fellow students talk about the difficulty of the exam and discuss their answers to certain questions.  The student decides not to think about the exam until her score arrives because "worrying isn't going to change the result."  She is delighted when she receives a very high score several weeks later.  Before receiving the result, the student exhibited which of the following defense mechanisms?

A)Denial
B)Dissociation
C)Intellectualization
D)Rationalization
E)Reaction formation
F)Repression
G)Suppression
Question
A 35-year-old woman comes to the office due to insomnia and fatigue over the past month.  She has felt increasingly depressed, irritable, and worthless since being let go from her job 5 weeks ago.  The week prior to her last menstrual period was particularly difficult, and she stayed in bed most of the day.  Over the last month, the patient has lost 3.6 kg (8 lb) and has felt unmotivated and unable to concentrate on looking for new work.  Other medical conditions include hypothyroidism and migraine headaches.  The patient drinks 1 or 2 glasses of wine a few times a week and smokes marijuana once a month.  Medications include levothyroxine and naproxen.  Physical examination is normal.  TSH level is 0.9 µU/mL.  Which of the following is the most likely diagnosis in this patient?

A)Adjustment disorder with depressed mood
B)Bipolar II disorder
C)Depressive disorder due to another medical condition
D)Major depressive disorder
E)Persistent depressive disorder
F)Premenstrual dysphoric disorder
G)Substance-induced depressive disorder
Question
A 24-year-old woman calls her obstetrician after giving birth to a healthy boy 5 days earlier.  She reports feeling depressed and irritable: "I thought I would be thrilled to have a baby, but then I felt really disappointed it was not a girl and find myself crying constantly."  The patient gets help from her sister during the day but is exhausted due to getting up at night to feed the baby.  She says, "My sister is so helpful, but I can't help snapping at her for no reason.  I get so worried that I won't be a decent mother."  The patient has no medical or psychiatric history, and the pregnancy and delivery were unremarkable.  She has no suicidal ideation or thoughts of harming the baby.  In addition to providing support, which of the following is the most appropriate response to the patient?

A)I am concerned that your depression may interfere with your ability to care for your baby right now.
B)I would like you to see a counselor for further evaluation of your mood.
C)Mood changes are normal after giving birth; we can follow up at your 6-week postpartum checkup.
D)Postpartum mood changes are common; please call me if you do not start to improve within the next week.
E)We should consider starting an antidepressant for postpartum depression.
Question
A 21-year-old woman, gravida 1 para 0, comes to the office for a prenatal visit at 16 weeks gestation.  The patient, who has not gained weight since her last visit 4 weeks ago, has occasional nausea with vomiting twice a week.  She has little appetite, which she attributes to mild indigestion, and her food consumption has decreased.  However, the patient craves ice and has been consuming it throughout the day for the last few months.  Medications include daily prenatal vitamins.  The patient was prescribed twice-daily iron for anemia but does not take it because it causes constipation.  She has no diarrhea, dysuria, chills, or fever.  She does not use tobacco.  Uterine fundus is consistent with a 16-week pregnancy.  Fetal heart tone is 140-149/min.  The patient's vital signs are within normal limits.  Which of the following is the most likely diagnosis?

A)Anorexia nervosa
B)Avoidant/restrictive food intake disorder
C)Bulimia nervosa
D)Hyperemesis gravidarum
E)Normal pregnancy craving
F)Pica
Question
A 35-year-old woman comes to the office due to worsening fatigue and tension headaches over the past year.  She describes always feeling "on edge" and lies awake at night worrying about various issues such as whether she is good at her job, whether her grown children like her, if she will get cancer one day, and whether her house will be broken into.  During the day she feels tired and her shoulders and neck ache; she has difficulty concentrating on her work and has become increasingly concerned about losing her job.  The patient has no medical or psychiatric history.  Physical examination and routine laboratory studies, including thyroid function tests, are normal.  The patient's anxiety has never been treated, but she once took a tablet of alprazolam offered by a friend.  This lowered her anxiety, but the effect did not last long.  The patient drinks a glass of wine on rare social occasions and does not use illicit drugs.  Which of the following is the most appropriate pharmacotherapy for this patient?

A)Alprazolam
B)Bupropion
C)Citalopram
D)Clonazepam
E)Diazepam
F)Propranolol
Question
A 62-year-old woman comes to the office for a checkup.  Her husband died 5 months ago in a biking accident, which she witnessed.  She has little appetite, resulting in weight loss of 3.17 kg (7 lb), and tends to wake up two hours before her alarm clock rings.  The patient feels overwhelmed at having to manage the household finances, saying, "My husband always took care of the bills so I wouldn't have to worry."  She avoids cycling, a hobby she shared with her husband, but continues to volunteer at a children's hospital.  The patient has no nightmares or suicidal thoughts.  During the visit, she tears up intermittently but smiles when sharing a memory of a vacation she took with her husband.  Which of the following is the best explanation for this patient's condition?

A)Dependent personality disorder
B)Major depressive disorder
C)Normal grief
D)Persistent complex bereavement disorder
E)Persistent depressive disorder
F)Post-traumatic stress disorder
Question
A 28-year-old woman, gravida 1 para 1, comes to the office for a 6-week postpartum checkup following an uncomplicated delivery.  The patient reports feeling increasingly fatigued and having little energy for the past few weeks.  She feels very anxious about being a new mother and has been getting up multiple times at night to check on the baby when he is sleeping.  She tends to skip meals and says, "I don't have time to sit and eat-I can't even find the time to shower."  Although the baby is healthy and doing well, the patient berates herself for being a "terrible mother."  She becomes tearful during the interview and says, "I don't know why I keep crying when I should be so happy."  Physical examination is normal.  The patient has no suicidal ideation or thoughts of harming the baby.  Which of the following is the most likely diagnosis?

A)Acute stress disorder
B)Adjustment disorder
C)Generalized anxiety disorder
D)Normal postpartum reaction
E)Postpartum depression
Question
A 5-year-old girl is brought to the office by her mother, who is concerned after observing her daughter speak to an empty chair in her room on several occasions.  The girl's medical history is significant for preterm birth at 36 weeks gestation but is otherwise noncontributory.  Her family history is significant for cerebrovascular accident and depression in her maternal grandfather and schizoaffective disorder in her father.  When questioned, the patient says she sees her grandmother at night and likes to speak to her before she goes to bed.  The grandmother died 2 months ago and had lived in the family home.  For a few weeks following the death, the girl would become tearful and frequently ask her parents when her grandmother would be coming back.  Which of the following is the most likely diagnosis in this patient?

A)Acute stress disorder
B)Brief psychotic disorder
C)Major depression with psychotic features
D)Normal behavior
E)Post-traumatic stress disorder
F)Schizophreniform disorder
Question
A 27-year-old man is brought to a family therapist by his wife following a violent outburst in which he nearly injured her.  They were having what seemed like a minor argument over a miscommunication about her being late when he suddenly flew into a rage, started shouting, and threw several plates against the wall.  His wife is now threatening to leave him because similar episodes keep happening despite his promise to control his anger.  The patient is remorseful and says, "I have been getting into trouble because of my temper since high school.  Once I get angry, I feel out of control and it's impossible to stop."  The patient has no medical history.  He drinks beer and uses cannabis to relax approximately 2-3 times a month.  Which of the following is the most likely diagnosis in this patient?

A)Antisocial personality disorder
B)Bipolar disorder, manic episode
C)Borderline personality disorder
D)Conduct disorder
E)Disruptive mood dysregulation disorder
F)Intermittent explosive disorder
G)Phencyclidine intoxication
Question
A 54-year-old man comes to the office due to concerns about having pancreatic cancer after a coworker died of the disease 6 months ago.  The patient has no epigastric pain, jaundice, or weight loss.  However, he worries constantly because in researching the illness he read that it may not have obvious symptoms in early stages and can be rapidly fatal.  The patient saw another physician 2 months ago, who performed a physical examination, laboratory evaluation, and abdominal CT scan.  The results were normal and the physician reassured the patient that he did not have cancer.  However, the patient reports that he has noticed occasional stomach noises after eating and would like to have additional testing done.  Which of the following is the most likely diagnosis?

A)Adjustment disorder with anxiety
B)Conversion disorder
C)Delusional disorder (somatic subtype)
D)Factitious disorder
E)Generalized anxiety disorder
F)Illness anxiety disorder
G)Somatic symptom disorder
Question
A 22-year-old college student comes to the office due to tension headaches and neck pain.  He has had these symptoms for many years, but they have increased in frequency over the past 7 months since he started working at a coffee shop. The patient attends college and says that although his grades are passing, he feels overwhelmed and fatigued: "I can't concentrate on my schoolwork and worry that I will flunk out of school and never get a good job."  He sleeps very fitfully and frequently worries about his grades, health, and social life.  He obsesses about minor comments his friends have made about how "serious" he is, and gets depressed thinking that he is not attractive enough to get a girlfriend.  Which of the following is the most likely diagnosis in this patient?

A)Adjustment disorder
B)Generalized anxiety disorder
C)Major depressive disorder
D)Obsessive-compulsive personality disorder
E)Social anxiety disorder
F)Somatic symptom disorder
Question
A 3-year-old boy is brought to the office for a well-child visit.  His mother reports that, apart from an ear infection last year, he is in good health.  The child started preschool this year.  He plays alongside other children and often copies what they are doing but does not play cooperatively with them.  The patient knows his age and gender and speaks in 3-word sentences.  He can ride a tricycle.  The patient cannot use a spoon or fork but enjoys eating with his hands.  He scribbles spontaneously but cannot copy a circle.  He weighs 14.5 kg (32 lb) and is 96.5 cm (3 ft 2 in) tall.  Head circumference is 50.8 cm (20 in).  Which of the following developmental milestones is likely delayed in this patient?

A)Cognitive
B)Fine motor
C)Gross motor
D)Language
E)Social
Question
A 5-year-old boy is brought to the office by his mother for a checkup.  He is healthy and about to start kindergarten.  His mother is concerned about his recent reaction to her brother's death.  "Although I told him that Uncle John died and can't be with us anymore, he insists that he is coming for his birthday and will take him to a ballgame.  He died over 3 months ago, but my son repeatedly asks me when he is coming back.  When I try to explain, he starts crying that his stomach hurts."  Physical examination is normal.  During the examination he tells the physician, "I try to clean up my toys before bedtime so mommy won't cry."  Which of the following is the most likely explanation for the patient's behavior?

A)Age-appropriate behavior
B)Complicated grief reaction
C)Developmental delay
D)Post-traumatic stress disorder
E)Regression
F)Somatic symptom disorder
Question
A 45-year-old woman comes to the office with multiple scratches, small sores, and abscesses on her left arm.  She is worried because the sores have been present for the past month and have not healed.  The patient has no idea how the sores developed and is concerned that they will spread.  She is a nurse who is well known to the staff from a similar presentation 2 years ago.  At that time, she had similar skin lesions, many of which were severely infected and did not respond to usual treatment.  Fecal bacteria were found in one of the sores.  On another occasion, the patient had a spreading infection and cellulitis that required a prolonged hospitalization and intravenous antibiotics.  She has no other medical problems.  Which of the following is the most likely cause of this patient's condition?

A)Borderline personality disorder
B)Conversion disorder
C)Factitious disorder
D)Illness anxiety disorder
E)Malingering
F)Somatic symptom disorder
Question
A 15-year-old boy is brought to the office by his parents due to concern about his development.  The patient has had a long-standing interest in fashion and a preference for having girls as friends, and his parents noticed that he recently bought women's underwear and shoes and went out to meet friends wearing makeup.  They worry that he is being bullied since moving to a new high school last year.  When interviewed privately the patient says, "I have always wanted to be a girl."  He is distressed that his voice is deepening and that he is developing facial and pubic hair, which he shaves regularly.  Which of the following is the most likely explanation of this patient's behavior?

A)Bisexuality
B)Body dysmorphic disorder
C)Fetishistic disorder
D)Gender dysphoria
E)Transvestic disorder
Question
A 29-year-old woman comes to the office due to persistent fatigue over the last 4 years.  She has also felt unhappy during this period, ever since being let go from her previous job.  The patient describes her fatigue as "having little energy to do things."  When asked what she enjoys, she replies that "everything in life is a chore" and that she feels hopeless that her life will improve.  The patient has no suicidal thoughts, problems with concentration, or changes in appetite or sleeping patterns.  She used marijuana as a teenager and drinks 1 or 2 glasses of wine on weekends.  Detailed workup, including urine toxicology screen, is negative.  Which of the following is the most likely diagnosis?

A)Adjustment disorder
B)Borderline personality disorder
C)Major depressive disorder
D)Persistent depressive disorder (dysthymia)
E)Substance-induced mood disorder
Question
A 14-year-old girl is brought to the office by her parents for a sports participation examination.  The patient is a competitive figure skater and practices before and after school 5 days a week.  Her parents are concerned because she recently announced that she will eat only a vegetarian diet, and they are having difficulty providing an adequate variety of meat-free food options around her busy school and athletic training schedule.  The patient is otherwise healthy; menarche occurred at age 12 and she has regular menstrual cycles.  Vital signs are normal.  BMI is 17 kg/m2.  Physical examination is normal for age.  Which of the following is the most appropriate next step in management of this patient?

A)Advise the patient to consume a protein supplement and multivitamin to support her athletic activity
B)Ask the patient to explain her reasons for wanting to change to a vegetarian diet
C)Explore the parents' beliefs regarding the benefits and risks of vegetarian diets
D)Provide education to the family regarding vegetarian meal planning
E)Reassure the parents that the patient is healthy, and no intervention is necessary at this time
Question
A 34-year-old man comes to the emergency department for evaluation of abdominal pain.  He reports severe pain unrelated to meals that is sharp, stabbing, and 10 out of 10 in severity.  The patient has been hospitalized twice in the past year for similar symptoms and states that a diagnosis was never found.  He declines when asked to sign a release of information to obtain his prior medical records.  Vital signs are within normal limits.  On physical examination, heart and lung sounds are normal.  Abdomen is soft and nondistended with normoactive bowel sounds.  The patient grimaces and moans in pain throughout the evaluation.  Laboratory evaluation and abdominal CT scan show no abnormalities.  After hearing the results, the patient asks for a note to excuse him from work so that he can recover at home for a few days.  Which of the following additional features is most likely to be present in this patient?

A)Chronic feelings of helplessness
B)Excessive anxiety about unexplained physical symptoms
C)Preoccupation with having a serious illness
D)Recurrent suicidal gestures or attempts
E)Refusal of invasive medical procedures
Question
A 22-year-old woman is brought to the emergency department for bizarre behavior over the past week.  Her parents state that she has been "talking nonsense" nonstop, laughing for no reason, and has not slept for several days.  During triage, the patient attempted to hit a staff member, resulting in administration of haloperidol.  While awaiting further evaluation, the parents alert the nurse that the patient is starting to "make weird faces" while sticking out her tongue.  Medical history is noncontributory, and the patient takes no medications.  Urine drug screen is positive for cannabis and opiates.  Which of the following is the most likely explanation for this patient's motor symptoms?

A)Catatonia
B)Conversion disorder
C)Medication reaction
D)Opioid withdrawal
E)Psychotic agitation
Question
A 26-year-old man is hospitalized after he was found in the park shouting and laughing to himself.  He insists on wearing a cap lined with several layers of aluminum foil and explains that the hat prevents laser beams from reprogramming his mind.  For the past 3 years, the patient has been hearing the voices of his deceased mother, the devil, and a world-famous singer.  He has had one previous psychiatric hospitalization, during which he responded well to haloperidol.  However, the patient stopped the medication shortly after discharge because he did not like the way it made him feel.  A decision is made to administer a second-generation antipsychotic medication.  Compared with first-generation antipsychotics, this class of medication is associated with which of the following?

A)Greater efficacy in the treatment of positive psychotic symptoms
B)Greater risk of anticholinergic effects
C)Greater risk of tardive dyskinesia
D)Lower risk of acute dystonia
E)Lower risk of metabolic adverse effects
F)Lower risk of seizures
Question
A 14-year-old boy is brought to the emergency department by a teacher after being found staggering and falling on the ground in the school parking lot between classes.  When she found him, the patient seemed lethargic and disoriented and his speech was slurred.  His condition slowly improved over the next 30 minutes; when the patient is evaluated in the emergency department an hour after the teacher found him, he is alert, oriented, and able to speak clearly.  Temperature is 36.7 C (98.1 F), blood pressure is 102/65 mm Hg, and pulse is 62/min.  On physical examination, there are abrasions on the right forearm and shin and a rash around the nostrils and mouth.  The patient has no known medical history.  Which of the following substances is the most likely cause of this patient's symptoms?

A)Alcohol
B)Benzodiazepines
C)Cocaine
D)Inhalants
E)Opiates
F)Phencyclidine
Question
A 17-year-old girl is brought to the office by her parents for evaluation following the accidental drowning of her younger sister a week ago.  The patient, her sister, and 2 friends had gone swimming in the ocean.  When the sister became submerged, the patient attempted to rescue her but was overpowered by strong rip currents and was forced to return to shore.  The mother is concerned that her daughter seems to be unaware of her sister's death and believes she is "in denial."  The patient was unable to answer questions from the police about the drowning and reports she cannot remember any details of what happened that day.  Physical examination is unremarkable.  On mental status examination, the patient is alert and fully oriented.  She becomes upset and confused when the incident is discussed.  Which of the following is the most likely diagnosis?

A)Adjustment disorder
B)Complicated grief reaction
C)Conversion disorder
D)Dissociative amnesia
E)Posttraumatic stress disorder
Question
A 55-year-old, previously healthy man is brought to the office by his wife after being forced into early retirement due to poor work performance.  The patient was a financial planner but began missing important deadlines and mismanaging his client's accounts 6 months ago.  He became more irritable during this time and started to curse at and insult his coworkers when they expressed concern about his performance.  The patient has also become verbally abusive toward his wife but appears indifferent to the hurt he causes.  She has had to take over the finances and grocery shopping.  She adds, "My husband has developed quite the sweet tooth.  He eats almost two boxes of cookies a day now."  Physical examination is unremarkable.  This patient is most likely to have which of the following neuropathologic findings?

A)Aggregations of phosphorylated tau protein
B)Cytoplasm inclusions with alpha synuclein
C)Cytosolic vacuolation of neurons and glia with prion inclusions
D)Extracellular deposition of beta-amyloid
E)Intracellular deposition of presenilin
Unlock Deck
Sign up to unlock the cards in this deck!
Unlock Deck
Unlock Deck
1/240
auto play flashcards
Play
simple tutorial
Full screen (f)
exit full mode
Deck 2: Behavioral science
1
A 46-year-old man is admitted to the hospital for atypical chest pain.  His medical history is significant for hypertension controlled with amlodipine and hypercholesterolemia treated with atorvastatin; he has a family history of depression and heart disease.  During review of his substance use history, the patient says that he has been a "regular drinker" for the last 5 years.  He drinks a 6-pack of beer every night and has 2 more beers in the morning to help him "get through the day."  He also admits to smoking marijuana occasionally but does not use tobacco or other illicit drugs.  Which of the following symptoms or signs is most likely to appear earliest during this patient's hospitalization?

A)Fluctuating arousal level
B)Hypersomnolence
C)Nystagmus
D)Tonic-clonic seizures
E)Tremulousness
F)Visual hallucinations
E
E   Ethanol use leads to a number of biochemical changes in the CNS.  It acutely potentiates the effects of GABA (the primary inhibitory neurotransmitter in the CNS) at GABA-A receptors, leading to sedation.  <strong>Chronic ethanol use</strong> causes downregulation of <strong>GABA receptors</strong>.  Alcohol also weakly inhibits excitatory NMDA receptors in the brain, and chronic exposure leads to upregulation of these receptors.  These adaptive changes result in tolerance (ie, the need to increase the dose to achieve the desirable effect) and symptoms of withdrawal on abrupt alcohol cessation, a common occurrence during hospital admission.  Both tolerance and withdrawal are signs of substance dependence. <strong>Withdrawal symptoms</strong> can begin as early as <strong>6 hours</strong> after the patient's last drink, typically reach maximal intensity in 2-3 days, and subside in 4-5 days if the withdrawal is not severe.  <strong>Tremor</strong>, or the shakes, is the most common <strong>initial finding</strong>.  Patients also typically have signs of mild autonomic dysfunction (eg, increased heart rate and respirations), gastrointestinal distress (eg, nausea, vomiting), and anxiety. <strong>(Choice A)</strong>  Fluctuating arousal levels are characteristic of delirium tremens, a potentially fatal manifestation of alcohol withdrawal that typically begins 48-96 hours after the last drink.  Other findings include sympathetic hyperactivity (eg, hyperthermia, hypertension), hallucinations (eg, visual, auditory, and/or tactile), and confusion. <strong>(Choices B and C)</strong>  Hypersomnolence and nystagmus can occur in acute ethanol intoxication but are not common during alcohol withdrawal.  Most patients have insomnia attributable to increased CNS excitability during the withdrawal period. <strong>(Choice D)</strong>  Tonic-clonic seizures can occur 12-48 hours after the last drink and affect <5% of patients undergoing alcohol withdrawal. <strong>(Choice F)</strong>  Alcoholic hallucinosis (ie, visual hallucinations) can develop within 12-48 hours after the last drink.  These early hallucinations are distinct from delirium tremens. <strong>Educational objective:</strong> Tremulousness is typically one of the earliest symptoms of alcohol withdrawal.  Other common symptoms include gastrointestinal distress, agitation, anxiety, and autonomic disturbance.  Delirium tremens is the most severe manifestation of alcohol withdrawal and typically begins 48-96 hours after the last drink. __________ References: The emergency medicine management of severe alcohol withdrawal. (http://www.ncbi.nlm.nih.gov/pubmed/28188055) Identification and management of alcohol withdrawal syndrome. (http://www.ncbi.nlm.nih.gov/pubmed/25666543)
Ethanol use leads to a number of biochemical changes in the CNS.  It acutely potentiates the effects of GABA (the primary inhibitory neurotransmitter in the CNS) at GABA-A receptors, leading to sedation.  Chronic ethanol use causes downregulation of GABA receptors.  Alcohol also weakly inhibits excitatory NMDA receptors in the brain, and chronic exposure leads to upregulation of these receptors.  These adaptive changes result in tolerance (ie, the need to increase the dose to achieve the desirable effect) and symptoms of withdrawal on abrupt alcohol cessation, a common occurrence during hospital admission.  Both tolerance and withdrawal are signs of substance dependence.
Withdrawal symptoms can begin as early as 6 hours after the patient's last drink, typically reach maximal intensity in 2-3 days, and subside in 4-5 days if the withdrawal is not severe.  Tremor, or "the shakes," is the most common initial finding.  Patients also typically have signs of mild autonomic dysfunction (eg, increased heart rate and respirations), gastrointestinal distress (eg, nausea, vomiting), and anxiety.
(Choice A)  Fluctuating arousal levels are characteristic of delirium tremens, a potentially fatal manifestation of alcohol withdrawal that typically begins 48-96 hours after the last drink.  Other findings include sympathetic hyperactivity (eg, hyperthermia, hypertension), hallucinations (eg, visual, auditory, and/or tactile), and confusion.
(Choices B and C)  Hypersomnolence and nystagmus can occur in acute ethanol intoxication but are not common during alcohol withdrawal.  Most patients have insomnia attributable to increased CNS excitability during the withdrawal period.
(Choice D)  Tonic-clonic seizures can occur 12-48 hours after the last drink and affect <5% of patients undergoing alcohol withdrawal.
(Choice F)  Alcoholic hallucinosis (ie, visual hallucinations) can develop within 12-48 hours after the last drink.  These early hallucinations are distinct from delirium tremens.
Educational objective:
Tremulousness is typically one of the earliest symptoms of alcohol withdrawal.  Other common symptoms include gastrointestinal distress, agitation, anxiety, and autonomic disturbance.  Delirium tremens is the most severe manifestation of alcohol withdrawal and typically begins 48-96 hours after the last drink.
__________
References:
The emergency medicine management of severe alcohol withdrawal.
(http://www.ncbi.nlm.nih.gov/pubmed/28188055)
Identification and management of alcohol withdrawal syndrome.
(http://www.ncbi.nlm.nih.gov/pubmed/25666543)
2
A 32-year-old man is diagnosed with major depressive disorder and started on the selective serotonin reuptake inhibitor sertraline.  At his 2-week follow-up, the patient reports that his mood is "about the same."  He continues to feel sad and unmotivated most days and is short-tempered with his wife and kids.  He has little interest in food, sleeps poorly, and struggles to stay focused at work.  In the first 2-3 days of taking the medication, the patient experienced some mild nausea and anxiety that has since resolved.  He is now tolerating the medication without difficulty but is considering stopping it because he is discouraged by the lack of clear improvement.  Which of the following is the most likely explanation for this patient's lack of response?

A)Comorbid anxiety disorder
B)Development of tolerance
C)Inadequate duration of treatment
D)Medication side effect
E)Poor treatment adherence
F)Treatment-resistant depression
C
This patient has classic symptoms of major depressive disorder (eg, depressed mood, loss of interest, sleep and appetite disturbance, impaired concentration) and is being treated appropriately with the selective serotonin reuptake inhibitor (SSRI) sertraline.  As a drug class, antidepressants are associated with an interval delay between drug initiation and clinical response.  At this point, the most likely explanation for this patient's poor response is inadequate duration of antidepressant treatment.
An adequate antidepressant trial is generally considered to be at least 4-6 weeks, so lack of significant improvement at the 2-week mark is not uncommon.  The patient should be encouraged to continue taking the medication for at least 2-4 more weeks before the next step in treatment is considered.
(Choices A and D)  Anxiety and nausea are common early side effects of SSRIs and typically resolve over time without intervention, as in this patient.  This patient is now tolerating the medication well.
(Choice B)  Tolerance refers to diminished response to a drug with repeated use.  This patient has not yet developed an effective therapeutic response.
(Choice E)  Difficulties with medication adherence should always be considered in a patient who is not responding to medication.  However, this patient discloses that he is thinking about stopping the medication; therefore, there is no reason to suspect that he has already done so.
(Choice F)  Treatment-resistant depression is commonly defined as failure to respond to at least 2 adequate antidepressant trials.  This patient cannot be considered treatment resistant because he has had an inadequate trial of a single medication.
Educational objective:
Antidepressants (eg, selective serotonin reuptake inhibitors) take 4-6 weeks to achieve maximal clinical effect.
__________
References:
Evaluating antidepressant treatment prior to adding second-line therapies among patients with treatment-resistant depression.
(http://www.ncbi.nlm.nih.gov/pubmed/26935957)
When should you move beyond first-line therapy for depression?
(http://www.ncbi.nlm.nih.gov/pubmed/20977871)
3
An 8-year-old boy is brought to the office due to severe behavioral problems.  The patient has been suspended from school on two occasions over the past year due to running around in the classroom and talking back to his teachers.  His parents report that he is "in constant motion" and are concerned about his poor grades and inability to follow directions or household routines.  The patient has a history of mild asthma and no other medical problems.  There is a family history of schizophrenia in a maternal uncle.  Physical examination shows marked fidgeting and difficulty staying seated but is otherwise normal.  The boy frequently interrupts his mother while she speaks with the physician.  Treatment with methylphenidate is recommended.  The parents should be educated about which of the following regarding this medication?

A)Acute dystonia
B)Decreased appetite and weight loss
C)Delayed onset of action
D)Polyuria and polydipsia
E)Increased appetite and weight gain
F)Initial worsening of symptoms
G)Life-threatening rash
H)Sedation
B
This patient's hyperactivity, impulsivity, and disorganization that are resulting in behavioral problems both at school and at home are characteristic of attention-deficit hyperactivity disorder (ADHD)Psychostimulants (methylphenidate and amphetamines) are first-line treatment for ADHD in school-age children and are generally safe and well tolerated.  They have a rapid onset of action, resulting in improvement in core ADHD symptoms for the expected duration of action (Choices C and F).
The most common adverse effects include decreased appetite, weight loss, and insomnia.  Most affected children will experience a mild decrease in appetite that can typically be managed by administering the medication after meals and encouraging the child to eat nutrient-dense foods.  Other psychostimulant adverse effects that are less common include tics and increases in heart rate and blood pressure.  During treatment, patients are typically seen monthly to monitor weight, height, heart rate, and blood pressure.
(Choices A, E, and H)  Increased appetite, weight gain, and sedation are adverse effects commonly associated with second-generation antipsychotics; acute dystonia is an extrapyramidal side effect most commonly seen with high-potency first-generation antipsychotics.  These drugs do not have a role in the treatment of ADHD.
(Choice D)  The mood stabilizer lithium has been associated with polyuria and polydipsia due to its potential to cause nephrogenic diabetes insipidus.  Lithium is used to treat bipolar disorder and is not effective in ADHD.
(Choice G)  The risk of life-threatening rash, such as seen in Stevens-Johnson syndrome, has been associated with lamotrigine, an anticonvulsant used to treat seizure and bipolar disorders.
Educational objective:
Decreased appetite and insomnia are the most common adverse effects of psychostimulant medications used to treat attention-deficit hyperactivity disorder.  They are usually mild and can be managed without stopping the medication.
__________
References:
Attention-deficit hyperactivity disorder medication use: factors involved in prescribing, safety aspects and outcomes.
(http://www.ncbi.nlm.nih.gov/pubmed/28382197)
Safety of medicines used for ADHD in children: a review of published prospective clinical trials.
(http://www.ncbi.nlm.nih.gov/pubmed/24748641)
4
A 61-year old woman comes to the office due to insomnia.  She has had difficulty falling asleep since her divorce was finalized 2 months ago and she relocated to a new state to be closer to her daughter and grandchildren.  The patient's sleep is restless and she is frequently awakened by household noises.  During this time, she has also felt anxious and tense.  The patient is overwhelmed by financial worries and the responsibilities of living on her own.  Although previously outgoing, she now avoids opportunities to meet new people and socializes only with her daughter.  The patient has no psychiatric history but does have a history of hypothyroidism treated with levothyroxine.  Physical examination is unremarkable.  TSH is 1.6 µU/mL.  Which of the following is the most likely diagnosis?

A)Acute stress disorder
B)Adjustment disorder
C)Anxiety disorder due to a medical condition
D)Generalized anxiety disorder
E)Medication-induced anxiety disorder
F)Normal stress response
Unlock Deck
Unlock for access to all 240 flashcards in this deck.
Unlock Deck
k this deck
5
A 35-year-old man comes to the office due to a lack of sexual interest.  He says, "I started seeing this woman 9 months ago and am really attracted to her.  We get along well and things are great, but I just don't want to have sex with her anymore."  On further questioning, the patient reluctantly admits that whenever they have intercourse he has an orgasm in less than a minute and finds this very embarrassing.  He does not have the same problem when he masturbates.  He says, "I'm worried that my girlfriend will leave me, and it's really affecting my self-esteem."  The patient has no other concerns.  His other medical conditions include type 1 diabetes mellitus, chronic insomnia, and a history of major depression.  His medications include insulin glargine, short-acting insulin, and trazodone.  He does not use illicit substances.  Vital signs are within normal limits.  Laboratory results are significant for hemoglobin A1c of 7% and fasting glucose of 130 mg/dL.  Which of the following is the most likely diagnosis?

A)Erectile disorder
B)Male hypoactive sexual desire disorder
C)Premature ejaculation
D)Sexual dysfunction due to diabetes
E)Sexual dysfunction due to major depressive disorder
F)Substance/medication-induced sexual dysfunction
Unlock Deck
Unlock for access to all 240 flashcards in this deck.
Unlock Deck
k this deck
6
An 82-year-old man is brought to the office by his daughter due to behavioral changes.  She reports that over the past year her father has become increasingly paranoid and frequently talks out loud as if in conversation when no one else is present.  The daughter says, "During a recent argument, he accused me of being an imposter and stealing his personal items.  Also, he used to be very clean, but his personal hygiene has gotten really bad.  He also stopped watching baseball, which used to be his favorite hobby."  The patient's medical history is significant for hypertension, hypercholesterolemia, and type 2 diabetes mellitus.  Medications include lisinopril, simvastatin, and metformin.  The patient has been unable to live independently for the past 2 years due to increasing forgetfulness and an inability to perform self-care.  Vital signs are stable, and physical examination and laboratory evaluation are unremarkable.  Which of the following is the most likely explanation for this patient's behavior?

A)Alzheimer disease
B)Delusional disorder
C)Major depression with psychotic features
D)Schizophrenia
E)Medication-induced psychotic disorder
Unlock Deck
Unlock for access to all 240 flashcards in this deck.
Unlock Deck
k this deck
7
A 23-year-old woman comes to the emergency department due to right wrist pain after a fall.  The patient fell onto her outstretched hand while walking across the floor at a gym where she exercises regularly.  She has no significant medical history and takes no medications.  The patient says she has tried several diets to lose weight and is currently consuming a vegan diet.  Her last menstrual period was 3 months ago.  She does not use tobacco, alcohol, or illicit drugs.  Temperature is 36.1 C (97 F), blood pressure is 90/58 mm Hg, and pulse is 50/min.  BMI is 18.3 kg/m2.  Weight is 50 kg (110.2 lb) and height is 165 cm (5 ft 5 in).  Physical examination shows tenderness and swelling over the distal radius.  The parotid glands are enlarged and there are fine, soft hairs on her extremities.  Urine pregnancy test is negative.  X-rays reveal a nondisplaced fracture of the right distal radius and generalized radiolucency of the bone.  Which of the following is the most likely diagnosis?

A)Anorexia nervosa
B)Avoidant/restrictive food intake disorder
C)Bulimia nervosa
D)Pellagra
E)Rickets
F)Scurvy
Unlock Deck
Unlock for access to all 240 flashcards in this deck.
Unlock Deck
k this deck
8
A 78-year-old man comes to the office for a regularly scheduled review of his chronic medical problems.  The patient has hypertension, coronary artery disease, and type 2 diabetes mellitus, for which he takes metformin, atorvastatin, lisinopril, and nitroglycerin as needed for chest pain.  He takes his medications as prescribed and follows diet and exercise instructions.  During the discussion, he hesitates, laughs nervously, and says, "I can't get an erection anymore, and my wife says I have to ask you about getting the 'blue pill.'"  Which of the following is the most appropriate response to this patient's concern?

A)"I can see that you feel uncomfortable talking about this.  It can be a sensitive subject for some men."
B)"I can understand your concern, but at your age, we hesitate to start too many medications."
C)"Medications for erectile dysfunction have significant side effects.  I would not pursue them unless you feel it is important."
D)"This is a very common problem for men as they age.  It is good that you mentioned it."
E)"We can try medication for erectile dysfunction, but it may not be effective at your age."
Unlock Deck
Unlock for access to all 240 flashcards in this deck.
Unlock Deck
k this deck
9
An 11-year-old girl is brought to the office due to disruptive behavior at home and at school.  Her parents report that she gets in trouble for talking during class and not following instructions.  Although the patient is of above-average intelligence, her grades are poor.  Teachers note that she makes careless mistakes while rushing through tests and frequently forgets to hand in assignments.  At home, she is easily distracted while trying to focus on her homework.  The patient has frequent conflicts with her mother, who says, "Getting her ready for school in the morning is impossible.  We're frequently late because she always misplaces her cell phone and books."  The patient has no other medical history, and developmental milestones are within normal range.  Physical examination shows no abnormalities.  The girl and her parents are willing to consider medication if it will help.  A drug with which of the following mechanisms of action is most appropriate for this patient?

A)Antagonism of alpha-2 adrenergic receptors
B)Antagonism of dopamine D2 receptors
C)Increased availability of norepinephrine and dopamine
D)Increased availability of serotonin
E)Increased availability of serotonin and norepinephrine
F)Positive allosteric modulation of GABA
Unlock Deck
Unlock for access to all 240 flashcards in this deck.
Unlock Deck
k this deck
10
A 40-year-old woman comes to the office due to worsening anxiety and insomnia over the past 3 months.  She says, "I'm really worried that something is wrong with me.  I was never a particularly anxious person, but now I feel anxious all the time.  Sometimes I feel panicky for no reason; my heart races and I break out in a sweat.  The only benefit is that I have lost 5 pounds (2.3 kg) without even trying."  The patient has no significant medical or psychiatric history.  She drinks 2-3 glasses of wine per week and does not smoke or use illicit drugs.  Blood pressure is 130/90 mm Hg and pulse is 112/min.  On physical examination, the patient is restless and has warm, moist skin and mild hand tremor bilaterally.  Mental status examination is notable for a frightened stare, anxious mood, and rapid speech.  Which of the following is the most likely diagnosis?

A)Alcohol withdrawal
B)Anxiety caused by a medical condition
C)Generalized anxiety disorder
D)Illness anxiety disorder
E)Panic disorder
F)Somatic symptom disorder
Unlock Deck
Unlock for access to all 240 flashcards in this deck.
Unlock Deck
k this deck
11
A 26-year-old woman comes to the emergency department to report that her life is in danger.  The patient believes that transmitters inserted in her bedroom walls are broadcasting her thoughts to a central government agency that is now plotting to kill her.  On mental status examination, she is fearful and paces the examining room.  The patient makes poor eye contact with the examiner, and her responses are interrupted by frequent pauses during which she appears to talk back to a person who is not present.  She has no medical history.  The patient smokes a pack of cigarettes daily and does not use illicit drugs.  Physical examination shows a thin, disheveled young woman but no abnormalities.  She is hospitalized, improves with medication over the course of 2 weeks, and is discharged on olanzapine.  At this patient's 3-month checkup, which of the following studies should be obtained?

A)Blood urea nitrogen and creatinine
B)Complete blood count
C)Electrocardiogram
D)Fasting glucose and lipid panel
E)Prolactin level
F)Thyroid function tests
Unlock Deck
Unlock for access to all 240 flashcards in this deck.
Unlock Deck
k this deck
12
A 16-year-old woman comes to the office due to malodorous vaginal discharge.  The patient arrives with her mother, who stays in the examination room for the evaluation.  The patient has had increased vaginal discharge for the past 2 days but no abnormal vaginal bleeding or abdominal or pelvic pain.  She is sexually active with a new partner and uses a progestin-releasing subdermal implant for contraception.  Her last menstrual period was 2 weeks ago.  She has no chronic medical conditions and takes no medications.  Vital signs are normal.  The patient appears anxious.  Abdominal examination shows no tenderness or palpable masses.  When the pelvic examination is attempted, the patient says that she is "embarrassed and anxious" and refuses the examination.  Which of the following is the most appropriate course of action?

A)Administer a benzodiazepine and proceed with the examination
B)Ask the patient's mother to leave the room and proceed with the examination
C)Distract the patient with questions while performing the examination
D)Do not perform the examination and ask the patient if she will perform self-collection for testing
E)Obtain permission from the patient's mother to proceed with the examination
Unlock Deck
Unlock for access to all 240 flashcards in this deck.
Unlock Deck
k this deck
13
A 32-year-old woman comes to the office due to long-standing anxiety.  She reports being excessively worried about everyday events and constantly feeling tense and unable to relax.  The patient is diagnosed with generalized anxiety disorder and treated with a selective serotonin reuptake inhibitor.  She returns to the office for a 2-month follow-up and reports that her overall anxiety has improved, but she continues to have periods of increased anxiety before bedtime that keep her awake at night.  The patient works as a 911 emergency dispatch operator and would be willing to add a medication at bedtime but needs to be "clearheaded" for work.  Which of the following medications is most appropriate for this patient's condition?

A)Chlordiazepoxide
B)Diazepam
C)Flurazepam
D)Lorazepam
E)Quetiapine
Unlock Deck
Unlock for access to all 240 flashcards in this deck.
Unlock Deck
k this deck
14
A 9-year-old boy is brought to the pediatrician due to poor school performance and difficulty making friends.  His parents say he always had "very high energy," but they noticed that this got worse after they moved 8 months ago, which they attributed to a normal adjustment period.  Over the past several months, however, the parents have received repeated phone calls from his teachers, reporting that he does not listen in class, has difficulty staying in his seat, turns in assignments late without his name, and frequently disrupts the class by talking out of turn.  At home the boy forgets to do his daily chores, regularly loses his books and homework, and delays the family by taking too long getting ready for school in the morning.  When his older brother teases him about this, the patient becomes irritable, and on a few occasions has shouted profanities and tried to hit his brother.  Which of the following is the most likely explanation for this patient's behavior?

A)Adjustment disorder
B)Attention-deficit hyperactivity disorder
C)Conduct disorder
D)Social anxiety disorder
E)Oppositional defiant disorder
Unlock Deck
Unlock for access to all 240 flashcards in this deck.
Unlock Deck
k this deck
15
A 22-year-old man comes to the emergency department due to sudden-onset pain and stiffness on one side of his neck.  He was diagnosed with a psychiatric disorder 2 days ago and has been adherent with his newly prescribed medication.  For the past few months, the patient has heard voices telling him to quit his job, and he believes that his girlfriend's parents are monitoring his sexual thoughts with a special device.  He has been socially withdrawn and has difficulty showering regularly.  Vital signs are within normal limits.  The head is tilted to one side and cannot be straightened without considerable pain.  The most likely cause of this patient's symptoms is antagonism of which of the following receptors?

A)Alpha-1 adrenergic
B)Dopaminergic D2
C)GABA-A
D)Histaminergic H1
E)Muscarinic M1
F)Serotonergic 5-HT2A
Unlock Deck
Unlock for access to all 240 flashcards in this deck.
Unlock Deck
k this deck
16
A 20-year-old college student comes to the office due to fatigue and decreased exercise tolerance.  She says, "I used to run 5 miles every day, but for the past month I get out of breath after running 3 miles.  I need to keep running so I don't get any fatter than I already am."  The patient also reports difficulty concentrating in class, depressed mood, constipation, and abdominal bloating.  Her appetite is normal, but she admits to occasionally forcing herself to vomit to prevent weight gain.  Physical examination is significant for dry skin, painless bilateral parotid swelling, pharyngeal erythema, and hypoactive bowel sounds.  Temperature is 36.2 C (97.2 F), blood pressure is 100/60 mm Hg, pulse is 62/min, and respirations are 16/min.  BMI is 17.5 kg/m2.  Which of the following is the most likely diagnosis?

A)Anorexia nervosa
B)Body dysmorphic disorder
C)Bulimia nervosa
D)Hypothyroidism
E)Irritable bowel syndrome
F)Major depressive disorder
Unlock Deck
Unlock for access to all 240 flashcards in this deck.
Unlock Deck
k this deck
17
A 46-year-old woman comes to the office due to sexual difficulties for the past year.  The patient rarely thinks about sex anymore and frequently declines sex when her husband tries to initiate.  She is attracted to her husband and worries that he assumes she is losing interest in him.  Although vaginal lubrication is adequate during sex and the patient can reach orgasm, she finds that her mind wanders during the experience.  She is unsure what is causing a decrease in her sex drive, although she notes that she received a promotion 3 months ago and has been working longer hours since then.  The patient describes her marriage as stable and loving.  She has a history of a depressive episode and currently takes bupropion.  Physical examination and laboratory studies, including thyroid function tests, show no abnormalities.  Which of the following is the most likely diagnosis?

A)Adjustment disorder
B)Antidepressant-induced sexual dysfunction
C)Female orgasmic disorder
D)Female sexual interest/arousal disorder
E)Genito-pelvic pain/penetration disorder
F)Normal menopausal changes
Unlock Deck
Unlock for access to all 240 flashcards in this deck.
Unlock Deck
k this deck
18
A 52-year-old woman comes to the office for a checkup.  Her medical problems include hypertension and hypercholesterolemia.  The patient has a long-standing relationship with her primary care physician, who has treated her children as well.  During the examination, she confides that she has been "down" since her youngest child left for college 2 months ago.  The patient is worried about her daughter being away from home for the first time and whether she will be successful at school.  At work, the patient occasionally has lapses of concentration when worrying about whether her daughter is okay, but it has not affected her productivity.  She says, "I still enjoy going out with my husband but all we do is talk about our kids."  The patient has occasional insomnia and tension headaches but notes that these are nothing new.  Physical examination is normal.  Which of the following is the most likely explanation for this patient's condition?

A)Adjustment disorder with depressed mood
B)Generalized anxiety disorder
C)Major depressive disorder
D)Normal sadness
E)Persistent depressive disorder
Unlock Deck
Unlock for access to all 240 flashcards in this deck.
Unlock Deck
k this deck
19
A 60-year-old woman comes to the office due to uncontrollable movements that have worsened.  Medical history is significant for hypertension, hyperlipidemia, osteoarthritis, and schizophrenia, and she has been on a stable medication regimen for years.  Throughout the interview, she exhibits facial grimacing, lip smacking, and twisting movements of her hands and feet.  The patient has lived in a group home for many years and worries that her housemates are avoiding her because her symptoms make her look "strange."  On mental status examination, the patient makes poor eye contact and is generally distrustful but has no specific delusions or hallucinations.  Which of the following is the most likely diagnosis?

A)Acute dystonia
B)Acute psychotic episode
C)Akathisia
D)Neuroleptic-induced parkinsonism
E)Neuroleptic malignant syndrome
F)Tardive dyskinesia
G)Tic disorder
Unlock Deck
Unlock for access to all 240 flashcards in this deck.
Unlock Deck
k this deck
20
A 63-year-old man comes to the office for follow-up of hypertension.  At his last appointment, he revealed that he had increased his intake of 12-oz cans of beer from 3 to 6 cans daily due to stress at work.  The physician advised him to seek help for his alcohol use because it could be causing his elevated blood pressure and has many negative health risks.  The patient now tells the physician, "I thought about what you said.  I know my alcohol use has gotten out of hand and is affecting my health.  My wife and daughter also say that I need to quit.  I have made an appointment with a counselor to talk about my options."  Which of the following best describes this patient's stage of behavioral change?

A)Action
B)Contemplation
C)Intellectualization
D)Precontemplation
E)Preparation
F)Rationalization
Unlock Deck
Unlock for access to all 240 flashcards in this deck.
Unlock Deck
k this deck
21
A 12-year-old boy is brought to the office due to behavioral problems.  His parents are upset about his poor grades and recent school suspension for setting fires in the locker room and stealing another student's cell phone.  They say that the boy has always been impulsive, had a lot of energy, and become angry and argumentative easily, but over the past year his aggressive behavior "has gotten out of control."  The patient says that the fires were an "accident," although he admits to being angry at the coach for kicking him off the basketball team.  Regarding the theft, he jokingly says, "People deserve to have their phones stolen when they are stupid enough to leave them in plain sight."  The patient has a history of getting into trouble for talking back to teachers, skipping class, and getting into fights since the fourth grade.  He has no medical history.  There is a family history of bipolar disorder and alcohol dependence.  The patient smokes cigarettes occasionally but denies illicit drug or alcohol use.  Which of the following is the most likely diagnosis in this patient?

A)Antisocial personality disorder
B)Attention-deficit hyperactivity disorder
C)Conduct disorder
D)Intermittent explosive disorder
E)Oppositional defiant disorder
F)Pyromania
Unlock Deck
Unlock for access to all 240 flashcards in this deck.
Unlock Deck
k this deck
22
A 28-year-old man is brought to the emergency department by his roommate, who is concerned about his change in behavior over the past 2 weeks.  The roommate describes the patient as "a regular guy who is usually very responsible."  Last week, the patient abruptly quit his job as a computer programmer and started placing large bets on an online gambling site because he was "sure to make millions."  The roommate says that the patient has been staying up most nights scribbling notes for his autobiography on small scraps of paper.  The patient says, "My new mission is to spread understanding."  He denies any alcohol or drug use, which his roommate affirms.  This patient is most likely to exhibit which of the following additional findings?

A)Flat affect
B)Low self-esteem
C)Poor hygiene
D)Pressured speech
E)Psychomotor slowing
F)Social withdrawal
G)Thought blocking
Unlock Deck
Unlock for access to all 240 flashcards in this deck.
Unlock Deck
k this deck
23
A 25-year-old graduate student with a history of recurrent ear infections as a child feels anxious and sweats when she is in the examination room with her primary care physician.  She recently got a roommate, a nursing student, who leaves her stethoscope on the coffee table after returning from class.  The patient sweats and feels her heart start to race whenever she sees the stethoscope.  This patient's response to her roommate's stethoscope is an example of which of the following phenomena?

A)Classical conditioning
B)Negative punishment
C)Negative reinforcement
D)Operant conditioning
E)Reaction formation
F)Regression
Unlock Deck
Unlock for access to all 240 flashcards in this deck.
Unlock Deck
k this deck
24
A 13-year-old boy is brought to the office due to anxiety and behavioral problems at school.  Although he has good grades, he frequently feels overwhelmed at school and his mother worries that he is socially awkward and has no friends.  The patient often appears anxious and uncomfortable in new social situations.  He insists on sitting in the same row and seat in every class and has had several inappropriate outbursts when told by teachers that this was not possible.  After school, the patient prefers to spend time at home.  His mother observes that he is obsessed with dates and comparing solar and lunar calendars and can discuss this for hours, but has difficulty shifting to other topics of conversation.  The patient was born at full term, and developmental milestones were within normal range.  Physical examination is unremarkable.  Which of the following is the most likely explanation of this patient's behavior?

A)Autism spectrum disorder
B)Obsessive compulsive disorder
C)Obsessive compulsive personality disorder
D)Schizoid personality disorder
E)Separation anxiety disorder
F)Social anxiety disorder
Unlock Deck
Unlock for access to all 240 flashcards in this deck.
Unlock Deck
k this deck
25
A 13-year-old girl is brought to the office for a routine visit.  Although the patient is doing well in school and has friends, her mother is concerned about a change in her behavior over the past several months.  They had previously enjoyed spending time together, but now the patient is frequently moody and spends most of her time on the computer in her locked bedroom.  The mother also observes that her daughter gets upset for no reason and has used swear words on occasion.  When the mother recently complimented her party outfit, the girl screamed, "I look hideous and I'm not going," and stormed out of the room.  Later that day, she seemed fine and went to the party.  On examination, the patient is pleasant and cooperative but makes little eye contact and asks if she can keep her tank top on during the examination.  Which of the following is the most likely explanation for this girl's behavior?

A)Body dysmorphic disorder
B)Borderline personality disorder
C)Cyclothymic disorder
D)Disruptive mood dysregulation disorder
E)Normal adolescent behavior
F)Oppositional defiant disorder
Unlock Deck
Unlock for access to all 240 flashcards in this deck.
Unlock Deck
k this deck
26
A 2-year-old boy is brought to the clinic by his mother, who is concerned about his language development.  She says, "I know that children speak at different ages and he is still young, but I am concerned that his speech isn't progressing and that he needs to interact with other children more.  His day care teacher says that he is shy and sits in a corner playing with his favorite truck.  We have also been busy with a recent move and have not given him as much attention, which doesn't help."  The patient's birth history, medical history, and audiology screening are unremarkable.  His mother is 6 months pregnant with her second child, and the family moved to a new home 3 months ago.  In the examination room, the patient sits in a corner, avoiding eye contact with the doctor and repeatedly spinning the wheels of a toy truck.  He does not respond to his name.  He makes occasional grunting sounds but says no fully formed words.  When his mother tries to take his truck away, he begins screaming and pulls it back.  Which of the following is the most likely diagnosis in this patient?

A)Adjustment disorder
B)Autism spectrum disorder
C)Language disorder
D)Normal variation in development
E)Selective mutism
F)Social anxiety disorder
G)Social (pragmatic) communication disorder
Unlock Deck
Unlock for access to all 240 flashcards in this deck.
Unlock Deck
k this deck
27
A 30-year-old man comes to the office due to depression.  He describes a 4-week history of severely depressed mood, loss of motivation, loss of interest, and sleeping 12 hours a day.  The patient has difficulty concentrating and worries that it is interfering with his work.  He is pessimistic about the future but has no suicidal thoughts.  The patient had a previous depressive episode in his 20s that responded rapidly to sertraline.  He felt "really good and energetic" after a few days of treatment but discontinued the antidepressant a week later as he did not feel that he needed it.  He drinks 1 or 2 beers on social occasions.  Physical examination is normal.  Which of the following would be most important to assess prior to initiating pharmacologic treatment for this patient?

A)Complete blood count
B)Comprehensive metabolic panel
C)Electrocardiogram
D)History of mania
E)History of sexual dysfunction
F)Urine toxicology
Unlock Deck
Unlock for access to all 240 flashcards in this deck.
Unlock Deck
k this deck
28
A 31-year-old man is brought to the emergency department by the police after he assaulted a stranger who he thought was following him.  The police report indicates that when the patient was arrested, he was shouting, "You don't understand, I'm the one who needs protection-the Russians are after my secrets!"  In the emergency department, the patient is diaphoretic and tremulous.  He is very easily distracted and cannot give a clear history.  Temperature is 37 C (98.6 F), blood pressure is 164/102 mm Hg, and pulse is 112/min with a normal rhythm.  Extraocular movements are intact, and the pupils are dilated.  His speech is loud, rapid, and difficult to interrupt.  Which of the following is the most likely diagnosis?

A)Anticholinergic toxicity
B)Bipolar disorder, manic episode
C)Cocaine intoxication
D)Delusional disorder, persecutory type
E)Opioid withdrawal
F)Phencyclidine intoxication
G)Schizophreniform disorder
Unlock Deck
Unlock for access to all 240 flashcards in this deck.
Unlock Deck
k this deck
29
A 35-year-old man comes to the office due to problems with irritability, anxiety, and low self-esteem.  He was recently fired due to poor work performance at his data entry job and worries about finding a new job and supporting himself.  The patient is currently in a relationship but mentions that his girlfriend is upset by his frequent lateness and forgetfulness and has threatened to leave him.  Further history indicates that his problems are long-standing and date back to childhood when he had behavioral problems in school.  The patient frequently feels overwhelmed due to being disorganized.  He frequently procrastinates, is bored easily at work, and jumps to another project before completing what he is working on.  The patient drinks 3 or 4 beers a week and smokes marijuana twice a month.  On mental status examination, he is cooperative and talkative but appears tense, restless, and easily distracted.  Which of the following is the most likely diagnosis in this patient?

A)Adjustment disorder
B)Attention-deficit hyperactivity disorder
C)Bipolar disorder
D)Borderline personality disorder
E)Generalized anxiety disorder
F)Persistent depressive disorder
G)Substance-induced mood disorder
Unlock Deck
Unlock for access to all 240 flashcards in this deck.
Unlock Deck
k this deck
30
A 27-year-old woman comes to the office due to concerns about her weight.  She is frustrated about gaining a few pounds, although she maintains a well-balanced diet and exercises for an hour 3 times a week.  The patient has been worried for years that her thighs are "huge" and "covered in cellulite."  She always wears loose-fitting pants and spends hours a day researching leg exercises and measuring the circumference of her thighs.  The patient generally stays at home and avoids social activities as she feels people tend to stare at her.  She weighs 58 kg (127.9 lb) and is 170 cm (5 ft 7 in) tall.  Physical examination is normal.  Which of the following is the most likely diagnosis?

A)Agoraphobia
B)Anorexia nervosa
C)Avoidant personality disorder
D)Body dysmorphic disorder
E)Bulimia nervosa
F)Delusional disorder
G)Obsessive-compulsive disorder
H)Social anxiety disorder
Unlock Deck
Unlock for access to all 240 flashcards in this deck.
Unlock Deck
k this deck
31
An 18-year-old boy is brought to the emergency department by his mother due to his strange behavior.  The mother says that he laughs inappropriately and seems sluggish.  The boy has recently been hanging out with a new group of friends that he describes as "really cool."  He has also stopped attending family events and appears apathetic about his schoolwork.  When asked in private, the boy admits smoking marijuana.  Which of the following physiologic effects is most likely to be observed in this patient?

A)Bradycardia
B)Conjunctival injection
C)Miosis
D)Mydriasis
E)Nystagmus
F)Respiratory depression
Unlock Deck
Unlock for access to all 240 flashcards in this deck.
Unlock Deck
k this deck
32
An 18-year-old woman comes to the office due to low energy, decreased appetite, insomnia, poor concentration, and decreased interest in her daily activities.  She has lost 3.6 kg (8 lb) over the last month.  The patient has no psychiatric history and has no current or significant medical illnesses.  Physical examination and laboratory evaluation are unremarkable.  Treatment with medication is initiated.  Three weeks later, the patient is brought to the emergency department by her mother for not sleeping at all for 3 nights.  The patient reports that she does not feel tired despite lack of sleep.  She spent a large amount of money on plane tickets and plans to travel around the world to promote world peace.  Her mother adds that she herself has suffered from severe mood swings in the past.  This patient was most likely started on which of the following medications 3 weeks ago?

A)Carbamazepine
B)Lamotrigine
C)Lithium
D)Olanzapine
E)Sertraline
F)Valproate
Unlock Deck
Unlock for access to all 240 flashcards in this deck.
Unlock Deck
k this deck
33
A 31-year-old woman comes to the office for a yearly checkup.  The patient has no medical conditions but expresses dissatisfaction with her appearance and wishes she could lose weight more easily.  She reports occasional constipation and fatigue.  The patient exercises daily and drinks 1 or 2 glasses of wine 2-3 times a week when socializing with friends.  Weight is 58.1 kg (128 lb) and height is 157.5 cm (5 ft 2 in).  BMI is 23.4 kg/m2.  Routine laboratory evaluation shows a potassium level of 3.1 mEq/L.  Physical examination is most likely to show which of the following abnormalities?

A)Bradycardia
B)Diaphoresis
C)Goiter
D)Hypertension
E)Lanugo
F)Malar rash
G)Parotid gland enlargement
Unlock Deck
Unlock for access to all 240 flashcards in this deck.
Unlock Deck
k this deck
34
A 21-year-old woman comes to the office due to constipation and vague abdominal pain.  She says, "For the last few months I have felt bloated and my stomach doesn't feel right."  Review of systems is notable for irregular menses and occasional fatigue.  Medical history is significant for a broken femur at age 6 but is otherwise noncontributory.  Family history is significant for Graves disease in her father and primary myelofibrosis in her paternal grandmother.  The patient smokes a pack of cigarettes daily and drinks wine when relaxing with friends.  She does not use illicit drugs.  Temperature is 37.2 C (99 F), blood pressure is 90/60 mm Hg, pulse is 118/min, and respirations are 16/min.  BMI is 25.6 kg/m2.  Physical examination shows eroded enamel of the teeth.Laboratory results are as follows: <strong>A 21-year-old woman comes to the office due to constipation and vague abdominal pain.  She says, For the last few months I have felt bloated and my stomach doesn't feel right.  Review of systems is notable for irregular menses and occasional fatigue.  Medical history is significant for a broken femur at age 6 but is otherwise noncontributory.  Family history is significant for Graves disease in her father and primary myelofibrosis in her paternal grandmother.  The patient smokes a pack of cigarettes daily and drinks wine when relaxing with friends.  She does not use illicit drugs.  Temperature is 37.2 C (99 F), blood pressure is 90/60 mm Hg, pulse is 118/min, and respirations are 16/min.  BMI is 25.6 kg/m<sup>2</sup>.  Physical examination shows eroded enamel of the teeth.Laboratory results are as follows:   Which of the following is the most likely cause of this patient's condition?</strong> A)Acute pancreatitis B)Alcohol use disorder C)Bulimia nervosa D)Hypothyroidism E)Irritable bowel syndrome F)Sjögren syndrome Which of the following is the most likely cause of this patient's condition?

A)Acute pancreatitis
B)Alcohol use disorder
C)Bulimia nervosa
D)Hypothyroidism
E)Irritable bowel syndrome
F)Sjögren syndrome
Unlock Deck
Unlock for access to all 240 flashcards in this deck.
Unlock Deck
k this deck
35
A 29-year-old woman comes to the office due to depression.  Since breaking up with her boyfriend last month, she has been extremely sad and has difficulty getting out of bed.  She describes sleeping 16 hours a day, increased appetite, a 4.5-kg (10-lb) weight gain, low energy, decreased concentration, and loss of interest in socializing with her friends and family.  The patient had 2 similar episodes at age 23 and 27.  She also describes brief periods in the past, lasting several days, when she was uncharacteristically confident and optimistic, successfully juggled 3 part-time jobs, and felt well rested and energetic despite sleeping only 3-4 hours a night.  The patient drinks a glass of wine several times a week but does not use tobacco or illicit drugs.  Which of the following is the most likely diagnosis?

A)Adjustment disorder with depressed mood
B)Bipolar I disorder
C)Bipolar II disorder
D)Cyclothymic disorder
E)Recurrent major depressive disorder
Unlock Deck
Unlock for access to all 240 flashcards in this deck.
Unlock Deck
k this deck
36
A healthy 8-year-old boy is brought to the office by his parents for an annual checkup.  Both parents express concern about their son's behavior.  The mother says, "He never listens and I am always worried that he is going to run out into the street without looking.  He rarely sits still and is always running and jumping on the furniture."  The father adds, "He talks all the time, interrupts me when I'm speaking, and, despite multiple reminders, forgets to do his chores and misplaces or loses his books or sporting equipment.  We thought he would grow out of it as he got older, but it seems to be getting worse."  Physical examination is unremarkable.  Which of the following would be most helpful in establishing the diagnosis?

A)Brain imaging
B)Hearing test
C)Quantitative EEG
D)Speech and language evaluation
E)Teacher evaluations
F)Toxicology screening
Unlock Deck
Unlock for access to all 240 flashcards in this deck.
Unlock Deck
k this deck
37
A 27-year-old man is brought to the emergency department by his wife.  She says that he has been "acting crazy" for the last 2 weeks.  He has hardly slept for the past 7 days and instead has worked on miscellaneous projects around the house.  The patient spent several thousand dollars on new power tools to accomplish these tasks.  When questioned, his speech is rapid and frenzied.  He feels "spectacular" and is creating an "architectural masterpiece."  The patient has had 2 previous depressive episodes.  Which of the following medications is the most appropriate agent for long-term management of this patient?

A)Bupropion
B)Chlorpromazine
C)Haloperidol
D)Lorazepam
E)Paroxetine
F)Valproate
Unlock Deck
Unlock for access to all 240 flashcards in this deck.
Unlock Deck
k this deck
38
A 28-year-old man is hospitalized due to the acute onset of severe depression and suicidal ideation for several days.  He was recently thrown out of his mother's home after stealing her money to support his drug habit.  The patient reports fatigue and vivid, disturbing dreams.  The nursing staff note that he is irritable, withdrawn, hypersomnolent, and hyperphagic.  The patient has an extensive history of substance abuse and has been hospitalized previously for alcohol detoxification.  Blood pressure is 110/80 mm Hg and pulse is 64/min.  Physical examination shows old injuries from a motorcycle accident but no other abnormalities.  On mental status examination, he appears lethargic and dysphoric.  This patient's current symptoms are most likely the result of which of the following conditions?

A)Alcohol withdrawal
B)Benzodiazepine withdrawal
C)Cannabis withdrawal
D)Cocaine withdrawal
E)Major depressive disorder
F)Opiate withdrawal
Unlock Deck
Unlock for access to all 240 flashcards in this deck.
Unlock Deck
k this deck
39
A 26-year-old woman comes to the office due to recent weight gain.  She has eaten more than usual over the last 5 months, has gained 3.2 kg (7 lb), and feels guilty and depressed about it.  Further questioning reveals that she consumes a large pizza and two large bags of chips in one sitting several times a week.  Afterward, the patient feels ashamed about being unable to control her intake and fasts to make up for it.  She is very distressed about being unable to lose weight despite exercising 2-3 hours a day.  Vital signs are within normal limits.  BMI is 23.7 kg/m2.  Despite being told that her BMI is normal, the patient insists that she is overweight.  Which of the following is the most likely diagnosis?

A)Adjustment disorder with depressed mood
B)Anorexia nervosa
C)Binge-eating disorder
D)Body dysmorphic disorder
E)Bulimia nervosa
F)Normal behavior
Unlock Deck
Unlock for access to all 240 flashcards in this deck.
Unlock Deck
k this deck
40
A 26-year-old man is brought to the emergency department by police after assaulting customers in a restaurant.  He explains that he meant no harm and was just trying to talk to people and touch them to "heal their pain."  The patient has been up all night for the past 10 days devising a global strategy to end world hunger and has written a hundred-page manifesto documenting his ideas.  Over this same period, he began hearing a voice telling him that he is "God's true son" and will need to sacrifice his life.  The patient has no psychiatric or medical history.  He drinks alcohol socially but does not use illicit drugs.  Physical examination is normal.  On mental status examination, the patient paces continuously.  His mood is very irritable, and his speech is loud, rapid, and difficult to interrupt.  Which of the following is the most likely diagnosis in this patient?

A)Bipolar I disorder with psychotic features
B)Bipolar II disorder
C)Brief psychotic disorder
D)Delusional disorder
E)Schizophreniform disorder
F)Substance-induced psychotic disorder
Unlock Deck
Unlock for access to all 240 flashcards in this deck.
Unlock Deck
k this deck
41
A 79-year-old man is brought to the office by his wife for evaluation of memory impairment.  The patient is a recently retired professor who began having memory problems a month ago.  He has been forgetting to take his medication, has had difficulty remembering the names of his grandchildren, and has neglected to turn off the stove on more than one occasion.  The patient used to enjoy reading and playing with his grandchildren but has recently stopped doing both of these activities.  He also has difficulty staying asleep, low appetite, and decreased energy.  The patient has hypertension and type 2 diabetes, for which he takes medications.  He has a family history of Alzheimer disease.  The patient says, "I just feel so worthless since retiring."  Temperature is 36.7 C (98.1 F), blood pressure is 119/78 mm Hg, pulse is 74/min, and respirations are 14/min.  Neurological examination reveals no focal deficits.  Montreal Cognitive Assessment score is 23 (normal: ≥26/30).  Which of the following is the most likely diagnosis?

A)Alzheimer disease
B)Dementia with Lewy bodies
C)Depression-related cognitive impairment
D)Frontotemporal dementia
E)Normal aging
F)Vascular dementia
Unlock Deck
Unlock for access to all 240 flashcards in this deck.
Unlock Deck
k this deck
42
A 28-year-old man comes to the office due to persistent fatigue.  For the past several weeks, he has been having trouble sleeping and is frequently late for work as he has difficulty getting out of bed.  At work, his mind wanders, and his supervisor has commented that the quality of his work has deteriorated.  The patient says, "I've been feeling down and don't feel like doing anything.  My appetite is poor.  I no longer go out with my friends and haven't gone to the gym in weeks."  The patient has no significant medical or psychiatric history.  He has 1 or 2 alcoholic drinks several days a week but does not use illicit drugs.  Routine laboratory evaluation, including thyroid function tests and serum B12 level, is normal.  Physical examination is unremarkable.  Mental status examination shows a cooperative man with sad affect.  No delusions are elicited.  The patient says he sometimes wishes he could "go to sleep and not wake up," but has no suicidal intent or plan.  First-line pharmacotherapy for this patient most likely involves a drug with which of the following mechanisms of action?

A)Antagonism of dopamine D2 receptors
B)Antagonism of serotonin 5-HT2 receptors
C)Inhibition of sodium channels
D)Inhibition of the serotonin transporter
E)Inhibition of monoamine oxidase
F)Potentiation of the effects of endogenous GABA
Unlock Deck
Unlock for access to all 240 flashcards in this deck.
Unlock Deck
k this deck
43
A 42-year-old man comes to the office for a checkup.  He has no medical symptoms but says, "I'm just feeling stressed and not like myself."  The patient is going through a contentious divorce.  He is outraged that his wife is pursuing full custody of their children and says, "I can't believe she is doing this to me."  His job as a legal researcher had been a welcome distraction from his upsetting personal life until yesterday when he was called to his supervisor's office for yelling at the receptionist when the copy machine ran out of ink.  Which of the following is the best explanation for this patient's behavior toward the receptionist?

A)Acting out
B)Denial
C)Displacement
D)Intellectualization
E)Reaction formation
F)Splitting
G)Sublimation
Unlock Deck
Unlock for access to all 240 flashcards in this deck.
Unlock Deck
k this deck
44
A 40-year-old woman is brought to the emergency department by her roommate due to significant left leg weakness.  The symptom began 3 days ago after her father had a heart attack.  There is no personal or family history of neurological disease; surgical history includes liposuction of the thighs and varicose vein removal.  She does not use tobacco, alcohol, or illicit drugs.  Temperature is 36.7 C (98.1 F), blood pressure is 123/81 mm Hg, pulse is 62/min, and respirations are 14/min.  Physical examination reveals symmetric 2+ deep tendon reflexes as well as normal muscle bulk and tone bilaterally in the upper and lower extremities.  Laboratory testing and neurologic imaging reveal no abnormalities.  Which of the following is the most likely diagnosis?

A)Body dysmorphic disorder
B)Conversion disorder
C)Factitious disorder
D)Illness anxiety disorder
E)Malingering
F)Somatic symptom disorder
Unlock Deck
Unlock for access to all 240 flashcards in this deck.
Unlock Deck
k this deck
45
A 72-year-old man is hospitalized due to a leg injury following a high-speed motor vehicle collision.  Medical history is significant for hypertension, hypercholesterolemia, and hearing loss.  The patient drinks a glass of wine every day.  Examination shows localized swelling and severe tenderness under the right knee; the presence of a tibial fracture is confirmed by imaging.  The patient undergoes open reduction and internal fixation without any perioperative complications.  On the first postoperative night, he describes visual hallucinations of his deceased brother in the hospital room with him and perseverates about people invading his home.  Vital signs are normal.  The patient is disoriented and unable to cooperate with the mental status examination.  He appears to respond to internal stimuli.  Which of the following interventions is most likely indicated to prevent further worsening of his condition?

A)Facilitating uninterrupted nocturnal sleep
B)Providing sedation by administering a benzodiazepine
C)Reducing daytime visual stimulation by closing window blinds
D)Reducing hyperstimulation by removing hearing aids
E)Treating pain with nonpharmacologic aids only
Unlock Deck
Unlock for access to all 240 flashcards in this deck.
Unlock Deck
k this deck
46
A 14-year-old boy is brought to the office by his parents, who are worried about his reaction to their recent decision to divorce.  Despite their efforts to be supportive and engage him, he has not expressed any feelings directly about the divorce.  When the patient is evaluated alone, he shares that he feels his parents are angry with him, although he cannot think of any instances when they expressed any anger toward him.  On examination, the patient appears sullen and reports his mood is "fine."  This patient is most likely using which of the following defense mechanisms?

A)Acting out
B)Displacement
C)Identification
D)Projection
E)Reaction formation
F)Regression
Unlock Deck
Unlock for access to all 240 flashcards in this deck.
Unlock Deck
k this deck
47
A 72-year-old retired man with end-stage renal disease becomes agitated during dialysis and attempts to abruptly leave in the middle of the procedure.  The patient accuses the staff of violating his rights and stealing his belongings but then appears to fall asleep.  Fifteen minutes later, he becomes violent and requires restraints.  Medical history is significant for hypertension, type 2 diabetes mellitus, hyperlipidemia, and peripheral vascular disease.  The patient has a history of major depression that has been in remission for 15 years.  His wife says that he has experienced mild memory and word-finding difficulties over the last year and that he often needs to make lists to remind himself of errands; however, she has never witnessed this type of agitated behavior.  Temperature is 38.3 C (100.9 F), blood pressure is 112/63 mm Hg, pulse is 93/min, and respirations are 18/min.  The patient has a left carotid bruit, and a dialysis catheter is present in the right internal jugular vein.  Bronchial breath sounds are heard at the right lung base.  During neurologic evaluation, he is confused but cooperative with no focal findings.  Which of the following is the most likely explanation for this patient's current behavior?

A)Alzheimer disease
B)Brief psychotic disorder
C)Delirium
D)Depression with psychotic features
E)Frontotemporal dementia
F)Vascular dementia
Unlock Deck
Unlock for access to all 240 flashcards in this deck.
Unlock Deck
k this deck
48
A 44-year-old man comes to the office due to low mood, impaired concentration, increased sleep and appetite, feelings of heaviness in his arms and legs, and loss of energy.  He is having difficulty at work as he is overly sensitive to criticism.  Although his boss has told him not to worry, the patient is concerned that his job is in jeopardy due to poor performance.  His symptoms started 8 months ago without any clear-cut precipitating event.  The patient received adequate trials of 3 different antidepressants without improvement and has been off of medication for the past several weeks.  His physician is now considering electroconvulsive therapy (ECT).  The patient declines ECT and asks to try another medication.  The physician then considers phenelzine.  The presence of which of the following additional symptoms would make the patient more likely to respond to this medication?

A)Auditory hallucinations
B)Decreased libido
C)Delusions
D)Memory impairment
E)Mood reactivity
F)Rapid speech
Unlock Deck
Unlock for access to all 240 flashcards in this deck.
Unlock Deck
k this deck
49
A 17-year-old girl is brought to the office by her parents due to recurrent episodes where "she seems to be lost in her own world."  The parents say that the episodes began about 2 years ago, but seem to be occurring more frequently since a difficult breakup with her boyfriend several months ago.  The patient acknowledges that she seems to "space out" easily.  She says, "I often feel numb, like I'm in a fog or a dream.  Sometimes, I feel as if I'm not myself, as if things are happening to someone else."  The patient uses nonsteroidal anti-inflammatory drugs for menstrual cramps.  She has no other medical conditions.  She smokes cigarettes but does not use alcohol or illicit drugs.  Physical and neurologic examinations are normal.  On mental status examination, the patient is cooperative with a mildly anxious mood and flat affect.  Which of the following is the most likely diagnosis?

A)Absence seizures
B)Brief psychotic disorder
C)Depersonalization/derealization disorder
D)Dissociative amnesia
E)Dissociative identity disorder
F)Post-traumatic stress disorder
Unlock Deck
Unlock for access to all 240 flashcards in this deck.
Unlock Deck
k this deck
50
A 45-year-old woman brings her stepson to the office for a well-child visit.  She never wanted children due to her unhappy childhood and wishes her marriage did not include being a stepmother to her husband's 5-year-old son.  In the waiting room, the woman seems excessively concerned when the boy accidentally trips and is clearly uninjured.  During the visit, she tells the physician that she adores the child and describes her elaborate preparations for his birthday party.  Which of the following defense mechanisms best explains this woman's behavior?

A)Displacement
B)Projection
C)Rationalization
D)Reaction formation
E)Splitting
F)Suppression
Unlock Deck
Unlock for access to all 240 flashcards in this deck.
Unlock Deck
k this deck
51
A 54-year-old man arrives for an appointment with a new physician.  At the front desk, the receptionist tells him that the physician is running behind schedule and gives him several forms to complete regarding his personal and family medical history.  The patient politely agrees to fill them out and has a seat in the waiting area.  However, when his paperwork is reviewed by the physician, it contains only the patient's signature.  The patient says, "I'm sure a doctor with your education can take a good history."  When asked if anything is wrong he says, "Oh, nothing, I'm fine."  Which of the following best describes this patient's behavior?

A)Acting out
B)Displacement
C)Passive aggression
D)Reaction formation
E)Splitting
F)Suppression
G)Undoing
Unlock Deck
Unlock for access to all 240 flashcards in this deck.
Unlock Deck
k this deck
52
A 63-year-old woman with a history of metastatic breast cancer comes to the office due to depressed mood.  Over the past month, the patient has become increasingly sad and frequently cries when thinking about her poor prognosis and dying.  She has lost 4.5 kg (10 lb) over the past month.  Her energy level is low, and she has difficulty falling asleep and frequent nighttime awakenings.  The patient has become very withdrawn, doesn't answer the phone, and no longer looks forward to family visits.  She feels bad about not wanting to be around her grandchildren.  On mental status examination, the patient is alert and oriented with depressed mood and affect.  She has no suicidal ideation.  Which of the following symptoms is most indicative of major depressive disorder in this patient?

A)Loss of interest in family
B)Low energy
C)Sleep disturbance
D)Thoughts of dying
E)Weight loss
Unlock Deck
Unlock for access to all 240 flashcards in this deck.
Unlock Deck
k this deck
53
A 71-year-old woman is brought to the emergency department with suicidal ideation.  Medical history includes major depressive disorder as well as chronic knee and back pain due to injuries sustained in a motor vehicle collision.  Developmental history is significant for physical abuse as a child.  Family history is significant for bipolar disorder and completed suicide in her father.  The patient reports increasing depression since losing her job a year ago and separating from her husband of 40 years 6 months ago.  She has a history of 2 prior hospitalizations for depression and a previous suicide attempt.  The patient is recovering from alcohol use disorder and attends weekly Alcoholics Anonymous meetings at her church.  Although she lives alone, she is close to her daughter, who lives nearby.  The patient reports that she owns a firearm that is kept in a locked cabinet.  Which of the following is the strongest risk factor for completed suicide in this patient?

A)Access to firearms
B)Age >70
C)Female sex
D)History of childhood trauma
E)History of completed suicide in a parent
F)History of suicide attempt
G)Major depressive disorder
H)Recent marital separation
Unlock Deck
Unlock for access to all 240 flashcards in this deck.
Unlock Deck
k this deck
54
A 48-year-old man begins psychotherapy due to depression and escalating conflicts with his boss.  He makes progress in the therapy sessions and reports feeling less depressed.  The patient has also gained insight into how his father was rarely available to him while growing up and recently told his father, "I am angry that you were never there for me."  The following week, the psychiatrist unexpectedly cancels a session to attend to an emergency.  At the next appointment, the patient says, "I feel as if you are not interested in listening to me, like I am not your priority."  Which of the following is the most likely explanation of this patient's attitude toward his psychiatrist?

A)Acting out
B)Displacement
C)Projection
D)Reaction formation
E)Regression
F)Transference
Unlock Deck
Unlock for access to all 240 flashcards in this deck.
Unlock Deck
k this deck
55
A 45-year-old man is hospitalized due to suicidal ideation.  Over the past month, the patient has become increasingly depressed, withdrawn, and physically restless.  He has had increasing difficulty concentrating at his job as a computer programmer.  The patient's appetite has been poor, and he has had difficulty falling and staying asleep.  He stopped going to work last week and refused to leave the house until his hospitalization.  The patient has no medical or psychiatric history.  Physical examination is unremarkable apart from a 4.5-kg (10-lb) weight loss.  On mental status examination, the patient appears depressed and anxious.  He reports that he is responsible for "all the evil in the world" and has heard a voice for the past week telling him that he does not deserve to live.  Which of the following is the most likely diagnosis in this patient?

A)Bipolar disorder with psychotic features
B)Delusional disorder
C)Major depressive disorder with psychotic features
D)Schizoaffective disorder
E)Schizophrenia
F)Schizophreniform disorder
Unlock Deck
Unlock for access to all 240 flashcards in this deck.
Unlock Deck
k this deck
56
A 72-year-old woman is admitted to the hospital due to abnormal vaginal bleeding.  Uterine cancer is diagnosed and the patient is scheduled for a hysterectomy.  Following successful surgery, the patient compliments her surgeon, exclaiming that she is "a brilliant doctor who saved my life."  After an uneventful recovery, the patient is ready to return home.  On the day of discharge, a nurse informs her that the surgeon is running late due to an emergency.  The patient responds angrily that the surgeon is "terrible and doesn't care about patients."  Which of the following is the most likely explanation for this patient's behavior?

A)Acting out
B)Displacement
C)Passive aggression
D)Projection
E)Reaction formation
F)Splitting
Unlock Deck
Unlock for access to all 240 flashcards in this deck.
Unlock Deck
k this deck
57
A resident physician is finishing up her shift when she receives a page that reads, "Patient in Room 121 asking to leave."  The patient was admitted 2 hours ago for worsening dyspnea.  On entering the room, the resident attempts to engage in a discussion about why the patient is requesting to leave.  In response, the patient pulls out his nasal cannula and breathlessly says, "You don't know anything; you're not even a real doctor!"  The resident is reminded of her father, who has frequently belittled her accomplishments.  She angrily informs the patient, "You can't leave and I'm ordering haloperidol to calm you down."  Which of the following best explains the resident's response to the patient?

A)Countertransference
B)Passive aggression
C)Projection
D)Reaction formation
E)Regression
F)Transference
Unlock Deck
Unlock for access to all 240 flashcards in this deck.
Unlock Deck
k this deck
58
A medical student just finished the USMLE Step 1 exam and is anxious about her performance.  She is especially unnerved when fellow students talk about the difficulty of the exam and discuss their answers to certain questions.  The student decides not to think about the exam until her score arrives because "worrying isn't going to change the result."  She is delighted when she receives a very high score several weeks later.  Before receiving the result, the student exhibited which of the following defense mechanisms?

A)Denial
B)Dissociation
C)Intellectualization
D)Rationalization
E)Reaction formation
F)Repression
G)Suppression
Unlock Deck
Unlock for access to all 240 flashcards in this deck.
Unlock Deck
k this deck
59
A 35-year-old woman comes to the office due to insomnia and fatigue over the past month.  She has felt increasingly depressed, irritable, and worthless since being let go from her job 5 weeks ago.  The week prior to her last menstrual period was particularly difficult, and she stayed in bed most of the day.  Over the last month, the patient has lost 3.6 kg (8 lb) and has felt unmotivated and unable to concentrate on looking for new work.  Other medical conditions include hypothyroidism and migraine headaches.  The patient drinks 1 or 2 glasses of wine a few times a week and smokes marijuana once a month.  Medications include levothyroxine and naproxen.  Physical examination is normal.  TSH level is 0.9 µU/mL.  Which of the following is the most likely diagnosis in this patient?

A)Adjustment disorder with depressed mood
B)Bipolar II disorder
C)Depressive disorder due to another medical condition
D)Major depressive disorder
E)Persistent depressive disorder
F)Premenstrual dysphoric disorder
G)Substance-induced depressive disorder
Unlock Deck
Unlock for access to all 240 flashcards in this deck.
Unlock Deck
k this deck
60
A 24-year-old woman calls her obstetrician after giving birth to a healthy boy 5 days earlier.  She reports feeling depressed and irritable: "I thought I would be thrilled to have a baby, but then I felt really disappointed it was not a girl and find myself crying constantly."  The patient gets help from her sister during the day but is exhausted due to getting up at night to feed the baby.  She says, "My sister is so helpful, but I can't help snapping at her for no reason.  I get so worried that I won't be a decent mother."  The patient has no medical or psychiatric history, and the pregnancy and delivery were unremarkable.  She has no suicidal ideation or thoughts of harming the baby.  In addition to providing support, which of the following is the most appropriate response to the patient?

A)I am concerned that your depression may interfere with your ability to care for your baby right now.
B)I would like you to see a counselor for further evaluation of your mood.
C)Mood changes are normal after giving birth; we can follow up at your 6-week postpartum checkup.
D)Postpartum mood changes are common; please call me if you do not start to improve within the next week.
E)We should consider starting an antidepressant for postpartum depression.
Unlock Deck
Unlock for access to all 240 flashcards in this deck.
Unlock Deck
k this deck
61
A 21-year-old woman, gravida 1 para 0, comes to the office for a prenatal visit at 16 weeks gestation.  The patient, who has not gained weight since her last visit 4 weeks ago, has occasional nausea with vomiting twice a week.  She has little appetite, which she attributes to mild indigestion, and her food consumption has decreased.  However, the patient craves ice and has been consuming it throughout the day for the last few months.  Medications include daily prenatal vitamins.  The patient was prescribed twice-daily iron for anemia but does not take it because it causes constipation.  She has no diarrhea, dysuria, chills, or fever.  She does not use tobacco.  Uterine fundus is consistent with a 16-week pregnancy.  Fetal heart tone is 140-149/min.  The patient's vital signs are within normal limits.  Which of the following is the most likely diagnosis?

A)Anorexia nervosa
B)Avoidant/restrictive food intake disorder
C)Bulimia nervosa
D)Hyperemesis gravidarum
E)Normal pregnancy craving
F)Pica
Unlock Deck
Unlock for access to all 240 flashcards in this deck.
Unlock Deck
k this deck
62
A 35-year-old woman comes to the office due to worsening fatigue and tension headaches over the past year.  She describes always feeling "on edge" and lies awake at night worrying about various issues such as whether she is good at her job, whether her grown children like her, if she will get cancer one day, and whether her house will be broken into.  During the day she feels tired and her shoulders and neck ache; she has difficulty concentrating on her work and has become increasingly concerned about losing her job.  The patient has no medical or psychiatric history.  Physical examination and routine laboratory studies, including thyroid function tests, are normal.  The patient's anxiety has never been treated, but she once took a tablet of alprazolam offered by a friend.  This lowered her anxiety, but the effect did not last long.  The patient drinks a glass of wine on rare social occasions and does not use illicit drugs.  Which of the following is the most appropriate pharmacotherapy for this patient?

A)Alprazolam
B)Bupropion
C)Citalopram
D)Clonazepam
E)Diazepam
F)Propranolol
Unlock Deck
Unlock for access to all 240 flashcards in this deck.
Unlock Deck
k this deck
63
A 62-year-old woman comes to the office for a checkup.  Her husband died 5 months ago in a biking accident, which she witnessed.  She has little appetite, resulting in weight loss of 3.17 kg (7 lb), and tends to wake up two hours before her alarm clock rings.  The patient feels overwhelmed at having to manage the household finances, saying, "My husband always took care of the bills so I wouldn't have to worry."  She avoids cycling, a hobby she shared with her husband, but continues to volunteer at a children's hospital.  The patient has no nightmares or suicidal thoughts.  During the visit, she tears up intermittently but smiles when sharing a memory of a vacation she took with her husband.  Which of the following is the best explanation for this patient's condition?

A)Dependent personality disorder
B)Major depressive disorder
C)Normal grief
D)Persistent complex bereavement disorder
E)Persistent depressive disorder
F)Post-traumatic stress disorder
Unlock Deck
Unlock for access to all 240 flashcards in this deck.
Unlock Deck
k this deck
64
A 28-year-old woman, gravida 1 para 1, comes to the office for a 6-week postpartum checkup following an uncomplicated delivery.  The patient reports feeling increasingly fatigued and having little energy for the past few weeks.  She feels very anxious about being a new mother and has been getting up multiple times at night to check on the baby when he is sleeping.  She tends to skip meals and says, "I don't have time to sit and eat-I can't even find the time to shower."  Although the baby is healthy and doing well, the patient berates herself for being a "terrible mother."  She becomes tearful during the interview and says, "I don't know why I keep crying when I should be so happy."  Physical examination is normal.  The patient has no suicidal ideation or thoughts of harming the baby.  Which of the following is the most likely diagnosis?

A)Acute stress disorder
B)Adjustment disorder
C)Generalized anxiety disorder
D)Normal postpartum reaction
E)Postpartum depression
Unlock Deck
Unlock for access to all 240 flashcards in this deck.
Unlock Deck
k this deck
65
A 5-year-old girl is brought to the office by her mother, who is concerned after observing her daughter speak to an empty chair in her room on several occasions.  The girl's medical history is significant for preterm birth at 36 weeks gestation but is otherwise noncontributory.  Her family history is significant for cerebrovascular accident and depression in her maternal grandfather and schizoaffective disorder in her father.  When questioned, the patient says she sees her grandmother at night and likes to speak to her before she goes to bed.  The grandmother died 2 months ago and had lived in the family home.  For a few weeks following the death, the girl would become tearful and frequently ask her parents when her grandmother would be coming back.  Which of the following is the most likely diagnosis in this patient?

A)Acute stress disorder
B)Brief psychotic disorder
C)Major depression with psychotic features
D)Normal behavior
E)Post-traumatic stress disorder
F)Schizophreniform disorder
Unlock Deck
Unlock for access to all 240 flashcards in this deck.
Unlock Deck
k this deck
66
A 27-year-old man is brought to a family therapist by his wife following a violent outburst in which he nearly injured her.  They were having what seemed like a minor argument over a miscommunication about her being late when he suddenly flew into a rage, started shouting, and threw several plates against the wall.  His wife is now threatening to leave him because similar episodes keep happening despite his promise to control his anger.  The patient is remorseful and says, "I have been getting into trouble because of my temper since high school.  Once I get angry, I feel out of control and it's impossible to stop."  The patient has no medical history.  He drinks beer and uses cannabis to relax approximately 2-3 times a month.  Which of the following is the most likely diagnosis in this patient?

A)Antisocial personality disorder
B)Bipolar disorder, manic episode
C)Borderline personality disorder
D)Conduct disorder
E)Disruptive mood dysregulation disorder
F)Intermittent explosive disorder
G)Phencyclidine intoxication
Unlock Deck
Unlock for access to all 240 flashcards in this deck.
Unlock Deck
k this deck
67
A 54-year-old man comes to the office due to concerns about having pancreatic cancer after a coworker died of the disease 6 months ago.  The patient has no epigastric pain, jaundice, or weight loss.  However, he worries constantly because in researching the illness he read that it may not have obvious symptoms in early stages and can be rapidly fatal.  The patient saw another physician 2 months ago, who performed a physical examination, laboratory evaluation, and abdominal CT scan.  The results were normal and the physician reassured the patient that he did not have cancer.  However, the patient reports that he has noticed occasional stomach noises after eating and would like to have additional testing done.  Which of the following is the most likely diagnosis?

A)Adjustment disorder with anxiety
B)Conversion disorder
C)Delusional disorder (somatic subtype)
D)Factitious disorder
E)Generalized anxiety disorder
F)Illness anxiety disorder
G)Somatic symptom disorder
Unlock Deck
Unlock for access to all 240 flashcards in this deck.
Unlock Deck
k this deck
68
A 22-year-old college student comes to the office due to tension headaches and neck pain.  He has had these symptoms for many years, but they have increased in frequency over the past 7 months since he started working at a coffee shop. The patient attends college and says that although his grades are passing, he feels overwhelmed and fatigued: "I can't concentrate on my schoolwork and worry that I will flunk out of school and never get a good job."  He sleeps very fitfully and frequently worries about his grades, health, and social life.  He obsesses about minor comments his friends have made about how "serious" he is, and gets depressed thinking that he is not attractive enough to get a girlfriend.  Which of the following is the most likely diagnosis in this patient?

A)Adjustment disorder
B)Generalized anxiety disorder
C)Major depressive disorder
D)Obsessive-compulsive personality disorder
E)Social anxiety disorder
F)Somatic symptom disorder
Unlock Deck
Unlock for access to all 240 flashcards in this deck.
Unlock Deck
k this deck
69
A 3-year-old boy is brought to the office for a well-child visit.  His mother reports that, apart from an ear infection last year, he is in good health.  The child started preschool this year.  He plays alongside other children and often copies what they are doing but does not play cooperatively with them.  The patient knows his age and gender and speaks in 3-word sentences.  He can ride a tricycle.  The patient cannot use a spoon or fork but enjoys eating with his hands.  He scribbles spontaneously but cannot copy a circle.  He weighs 14.5 kg (32 lb) and is 96.5 cm (3 ft 2 in) tall.  Head circumference is 50.8 cm (20 in).  Which of the following developmental milestones is likely delayed in this patient?

A)Cognitive
B)Fine motor
C)Gross motor
D)Language
E)Social
Unlock Deck
Unlock for access to all 240 flashcards in this deck.
Unlock Deck
k this deck
70
A 5-year-old boy is brought to the office by his mother for a checkup.  He is healthy and about to start kindergarten.  His mother is concerned about his recent reaction to her brother's death.  "Although I told him that Uncle John died and can't be with us anymore, he insists that he is coming for his birthday and will take him to a ballgame.  He died over 3 months ago, but my son repeatedly asks me when he is coming back.  When I try to explain, he starts crying that his stomach hurts."  Physical examination is normal.  During the examination he tells the physician, "I try to clean up my toys before bedtime so mommy won't cry."  Which of the following is the most likely explanation for the patient's behavior?

A)Age-appropriate behavior
B)Complicated grief reaction
C)Developmental delay
D)Post-traumatic stress disorder
E)Regression
F)Somatic symptom disorder
Unlock Deck
Unlock for access to all 240 flashcards in this deck.
Unlock Deck
k this deck
71
A 45-year-old woman comes to the office with multiple scratches, small sores, and abscesses on her left arm.  She is worried because the sores have been present for the past month and have not healed.  The patient has no idea how the sores developed and is concerned that they will spread.  She is a nurse who is well known to the staff from a similar presentation 2 years ago.  At that time, she had similar skin lesions, many of which were severely infected and did not respond to usual treatment.  Fecal bacteria were found in one of the sores.  On another occasion, the patient had a spreading infection and cellulitis that required a prolonged hospitalization and intravenous antibiotics.  She has no other medical problems.  Which of the following is the most likely cause of this patient's condition?

A)Borderline personality disorder
B)Conversion disorder
C)Factitious disorder
D)Illness anxiety disorder
E)Malingering
F)Somatic symptom disorder
Unlock Deck
Unlock for access to all 240 flashcards in this deck.
Unlock Deck
k this deck
72
A 15-year-old boy is brought to the office by his parents due to concern about his development.  The patient has had a long-standing interest in fashion and a preference for having girls as friends, and his parents noticed that he recently bought women's underwear and shoes and went out to meet friends wearing makeup.  They worry that he is being bullied since moving to a new high school last year.  When interviewed privately the patient says, "I have always wanted to be a girl."  He is distressed that his voice is deepening and that he is developing facial and pubic hair, which he shaves regularly.  Which of the following is the most likely explanation of this patient's behavior?

A)Bisexuality
B)Body dysmorphic disorder
C)Fetishistic disorder
D)Gender dysphoria
E)Transvestic disorder
Unlock Deck
Unlock for access to all 240 flashcards in this deck.
Unlock Deck
k this deck
73
A 29-year-old woman comes to the office due to persistent fatigue over the last 4 years.  She has also felt unhappy during this period, ever since being let go from her previous job.  The patient describes her fatigue as "having little energy to do things."  When asked what she enjoys, she replies that "everything in life is a chore" and that she feels hopeless that her life will improve.  The patient has no suicidal thoughts, problems with concentration, or changes in appetite or sleeping patterns.  She used marijuana as a teenager and drinks 1 or 2 glasses of wine on weekends.  Detailed workup, including urine toxicology screen, is negative.  Which of the following is the most likely diagnosis?

A)Adjustment disorder
B)Borderline personality disorder
C)Major depressive disorder
D)Persistent depressive disorder (dysthymia)
E)Substance-induced mood disorder
Unlock Deck
Unlock for access to all 240 flashcards in this deck.
Unlock Deck
k this deck
74
A 14-year-old girl is brought to the office by her parents for a sports participation examination.  The patient is a competitive figure skater and practices before and after school 5 days a week.  Her parents are concerned because she recently announced that she will eat only a vegetarian diet, and they are having difficulty providing an adequate variety of meat-free food options around her busy school and athletic training schedule.  The patient is otherwise healthy; menarche occurred at age 12 and she has regular menstrual cycles.  Vital signs are normal.  BMI is 17 kg/m2.  Physical examination is normal for age.  Which of the following is the most appropriate next step in management of this patient?

A)Advise the patient to consume a protein supplement and multivitamin to support her athletic activity
B)Ask the patient to explain her reasons for wanting to change to a vegetarian diet
C)Explore the parents' beliefs regarding the benefits and risks of vegetarian diets
D)Provide education to the family regarding vegetarian meal planning
E)Reassure the parents that the patient is healthy, and no intervention is necessary at this time
Unlock Deck
Unlock for access to all 240 flashcards in this deck.
Unlock Deck
k this deck
75
A 34-year-old man comes to the emergency department for evaluation of abdominal pain.  He reports severe pain unrelated to meals that is sharp, stabbing, and 10 out of 10 in severity.  The patient has been hospitalized twice in the past year for similar symptoms and states that a diagnosis was never found.  He declines when asked to sign a release of information to obtain his prior medical records.  Vital signs are within normal limits.  On physical examination, heart and lung sounds are normal.  Abdomen is soft and nondistended with normoactive bowel sounds.  The patient grimaces and moans in pain throughout the evaluation.  Laboratory evaluation and abdominal CT scan show no abnormalities.  After hearing the results, the patient asks for a note to excuse him from work so that he can recover at home for a few days.  Which of the following additional features is most likely to be present in this patient?

A)Chronic feelings of helplessness
B)Excessive anxiety about unexplained physical symptoms
C)Preoccupation with having a serious illness
D)Recurrent suicidal gestures or attempts
E)Refusal of invasive medical procedures
Unlock Deck
Unlock for access to all 240 flashcards in this deck.
Unlock Deck
k this deck
76
A 22-year-old woman is brought to the emergency department for bizarre behavior over the past week.  Her parents state that she has been "talking nonsense" nonstop, laughing for no reason, and has not slept for several days.  During triage, the patient attempted to hit a staff member, resulting in administration of haloperidol.  While awaiting further evaluation, the parents alert the nurse that the patient is starting to "make weird faces" while sticking out her tongue.  Medical history is noncontributory, and the patient takes no medications.  Urine drug screen is positive for cannabis and opiates.  Which of the following is the most likely explanation for this patient's motor symptoms?

A)Catatonia
B)Conversion disorder
C)Medication reaction
D)Opioid withdrawal
E)Psychotic agitation
Unlock Deck
Unlock for access to all 240 flashcards in this deck.
Unlock Deck
k this deck
77
A 26-year-old man is hospitalized after he was found in the park shouting and laughing to himself.  He insists on wearing a cap lined with several layers of aluminum foil and explains that the hat prevents laser beams from reprogramming his mind.  For the past 3 years, the patient has been hearing the voices of his deceased mother, the devil, and a world-famous singer.  He has had one previous psychiatric hospitalization, during which he responded well to haloperidol.  However, the patient stopped the medication shortly after discharge because he did not like the way it made him feel.  A decision is made to administer a second-generation antipsychotic medication.  Compared with first-generation antipsychotics, this class of medication is associated with which of the following?

A)Greater efficacy in the treatment of positive psychotic symptoms
B)Greater risk of anticholinergic effects
C)Greater risk of tardive dyskinesia
D)Lower risk of acute dystonia
E)Lower risk of metabolic adverse effects
F)Lower risk of seizures
Unlock Deck
Unlock for access to all 240 flashcards in this deck.
Unlock Deck
k this deck
78
A 14-year-old boy is brought to the emergency department by a teacher after being found staggering and falling on the ground in the school parking lot between classes.  When she found him, the patient seemed lethargic and disoriented and his speech was slurred.  His condition slowly improved over the next 30 minutes; when the patient is evaluated in the emergency department an hour after the teacher found him, he is alert, oriented, and able to speak clearly.  Temperature is 36.7 C (98.1 F), blood pressure is 102/65 mm Hg, and pulse is 62/min.  On physical examination, there are abrasions on the right forearm and shin and a rash around the nostrils and mouth.  The patient has no known medical history.  Which of the following substances is the most likely cause of this patient's symptoms?

A)Alcohol
B)Benzodiazepines
C)Cocaine
D)Inhalants
E)Opiates
F)Phencyclidine
Unlock Deck
Unlock for access to all 240 flashcards in this deck.
Unlock Deck
k this deck
79
A 17-year-old girl is brought to the office by her parents for evaluation following the accidental drowning of her younger sister a week ago.  The patient, her sister, and 2 friends had gone swimming in the ocean.  When the sister became submerged, the patient attempted to rescue her but was overpowered by strong rip currents and was forced to return to shore.  The mother is concerned that her daughter seems to be unaware of her sister's death and believes she is "in denial."  The patient was unable to answer questions from the police about the drowning and reports she cannot remember any details of what happened that day.  Physical examination is unremarkable.  On mental status examination, the patient is alert and fully oriented.  She becomes upset and confused when the incident is discussed.  Which of the following is the most likely diagnosis?

A)Adjustment disorder
B)Complicated grief reaction
C)Conversion disorder
D)Dissociative amnesia
E)Posttraumatic stress disorder
Unlock Deck
Unlock for access to all 240 flashcards in this deck.
Unlock Deck
k this deck
80
A 55-year-old, previously healthy man is brought to the office by his wife after being forced into early retirement due to poor work performance.  The patient was a financial planner but began missing important deadlines and mismanaging his client's accounts 6 months ago.  He became more irritable during this time and started to curse at and insult his coworkers when they expressed concern about his performance.  The patient has also become verbally abusive toward his wife but appears indifferent to the hurt he causes.  She has had to take over the finances and grocery shopping.  She adds, "My husband has developed quite the sweet tooth.  He eats almost two boxes of cookies a day now."  Physical examination is unremarkable.  This patient is most likely to have which of the following neuropathologic findings?

A)Aggregations of phosphorylated tau protein
B)Cytoplasm inclusions with alpha synuclein
C)Cytosolic vacuolation of neurons and glia with prion inclusions
D)Extracellular deposition of beta-amyloid
E)Intracellular deposition of presenilin
Unlock Deck
Unlock for access to all 240 flashcards in this deck.
Unlock Deck
k this deck
locked card icon
Unlock Deck
Unlock for access to all 240 flashcards in this deck.